NIFT 2024 MFM Question Paper (Available Soon)- Download Answer Key with Solutions PDF

Chanpreet Kaur's profile photo

Chanpreet Kaur

Content Writer | MBA Professional | Updated 3+ months ago

The NIFT 2024 Master of Fashion Management (MFM) Entrance Exam is a national-level test conducted by the National Institute of Fashion Technology (NIFT) for admission to its postgraduate design program. The exam assesses candidates on their creativity, analytical thinking, problem-solving ability, and knowledge of design trends.The NIFT 2024 Master of Fashion Management (MFM) Entrance Exam has a total of 150 marks and a duration of 3 hours. NIFT Master of Fashion Management (MFM) Question Paper With Solution PDF is available here.

NIFT 2024 MFM Question Paper With Answer Key

NIFT 2024 MFM Question Paper With Answer Key download iconDownload Check Solutions

NIFT 2024 MFM Question with Solutions


Question 1:

Choose the appropriate idiom to fill in the blank:

On arrival of the police the agitators ran \hspace{3cm}

  • (A) kith and kin
  • (B) helter skelter
  • (C) loaves and fish
  • (D) rank and file
Correct Answer: (2) helter skelter
View Solution

The phrase "helter skelter" refers to a disorganized or chaotic state, which fits the situation described in the sentence where the agitators ran in a panic upon the arrival of the police. This idiom conveys the image of confusion and disorder, which is the appropriate response in this context.

The other options are not suitable in this context:

- "kith and kin" refers to one’s family and relatives, which doesn’t fit the sentence.

- "loaves and fish" refers to abundance or miracles, not suitable here.

- "rank and file" refers to ordinary members of an organization, which doesn’t apply to how the agitators reacted. Quick Tip: When choosing the correct idiom, consider the context and meaning of the phrase. In cases of confusion or chaos, idioms like "helter skelter" are appropriate.


Question 2:

Correct the given sentence:

He had rather starved than beg.

  • (A) He had rather to starve than beg.
  • (B) He had rather starve than beg.
  • (C) He has rather starved than begged.
  • (D) He had rather starved than begged.
Correct Answer: (4) He had rather starved than begged.
View Solution

The sentence "He had rather starved than beg" is incorrect because after the word "rather," the verb that follows should be in the base form (not in the past tense). The correct structure should use the past perfect tense "had rather" followed by the base form of the verb. The correct option is therefore "He had rather starved than begged."

The other options are incorrect for the following reasons:

- "He had rather to starve than beg" is incorrect because "rather" is not followed by "to."

- "He had rather starve than beg" doesn't convey the intended comparison in the past perfect tense.

- "He has rather starved than begged" uses "has" instead of "had," which is grammatically incorrect here.
Quick Tip: When using "had rather," ensure that it is followed by the base form of the verb and the correct tense is used to maintain proper structure in the sentence.


Question 3:

Correct the given sentence:

Either the teacher or the students have left his classroom unlocked.

  • (A) Either the teacher or the students has left his classroom unlocked.
  • (B) Either the teacher or the students have left his classroom unlocked.
  • (C) Either the teacher or the students have left their classroom unlocked.
  • (D) Either the teacher or the students has left their classroom unlocked.
  • (b) Incorrect: Uses "his," which does not agree with "students."
  • (c) Correct: Uses "have" (matching "students") and "their" (matching "students").
Correct Answer: (C) Either the teacher or the students have left their classroom unlocked.
View Solution

Step 1: Subject-Verb Agreement in "Either... or..."

In constructions with "either... or..." the verb agrees with the subject closest to it. Since "students" is plural, the verb should be "have" to maintain agreement.

Step 2: Correct Pronoun Usage

The pronoun "his" is incorrect because it does not match "students," which is plural. The correct pronoun is "their" to reflect agreement with the plural noun.

Step 3: Evaluating the Options

(a) Incorrect: Uses "has," which does not agree with "students."

% Option
(b) Incorrect: Uses "his," which does not agree with "students."

% Option
(c) Correct: Uses "have" (matching "students") and "their" (matching "students").

% Option
(d) Incorrect: Uses "has," which does not agree with "students."



Final Answer:
The correct sentence is: \[ "Either the teacher or the students have left their classroom unlocked." \] Quick Tip: In "either... or..." constructions, make sure the verb agrees with the closest subject, and use the correct pronoun to match the antecedent in number and gender.


Question 4:


Choose the word with nearly the same meaning as the given word:

INTERVENTION

  • (A) interception
  • (B) interruption
  • (C) mediation
  • (D) meddling
Correct Answer: (C) mediation
View Solution

The word "intervention" refers to the action of intervening, often with the intent to resolve a conflict or come to a decision. "Mediation" is the closest in meaning as it involves intervention in a dispute in order to resolve it or bring about a settlement.

The other options are incorrect for the following reasons:

- "Interception" refers to the act of intercepting, such as catching or seizing something on its way from one place to another, which is different from intervention.

- "Interruption" refers to the act of stopping the continuous progress of an activity or process, not necessarily involving resolution.

- "Meddling" implies interfering in something that is not one's concern, often in a way that is annoying or damaging, which differs from the constructive or resolving intent of intervention.
Quick Tip: When matching synonyms, focus on the context in which the words are often used to ensure the chosen word reflects a similar intent or outcome as the original.


Question 5:


Choose the word with nearly the same meaning as the given word:

INSOLVENT

  • (A) flourishing
  • (B) bankrupt
  • (C) soluble
  • (D) opprobrious
Correct Answer: (2) bankrupt
View Solution

"Insolvent" refers to the inability to pay debts owed, which aligns directly with the definition of "bankrupt." A bankrupt person or entity is one that has been legally declared unable to pay outstanding debts.

The other options are incorrect for the following reasons:

- "Flourishing" means to grow or develop in a healthy or vigorous way, which is the opposite of insolvent.

- "Soluble" pertains to the ability of a substance to dissolve, typically in a liquid, and is unrelated to financial solvency.

- "Opprobrious" relates to expressing scorn or criticism, which does not align with the financial context of being insolvent.
Quick Tip: Understanding the precise definitions of terms like "insolvent" helps in accurately matching them with their synonyms, especially in contexts involving legal or financial conditions.


Question 6:


Choose the word with nearly the same meaning as the given word:

LAUNCH

  • (A) review
  • (B) begin
  • (C) propel
  • (D) force
Correct Answer: (2) begin
View Solution

The term "launch" generally refers to the act of starting or initiating something, such as a project or a product. Thus, the word "begin" is the closest in meaning, as it also conveys the start of an action or process.

The other options are incorrect for the following reasons:

- "Review" implies examining something already in existence, which does not correspond to the initiating context of "launch."

- "Propel" suggests pushing or driving an object forward, which, while related to the action following a launch, does not directly convey the act of initiating.

- "Force" implies applying power or coercion, which differs from the voluntary initiation denoted by "launch."
Quick Tip: When matching synonyms, consider the primary action or essence of the word to find a term that captures the same meaning in similar contexts.


Question 7:


Give one word for the underlined expression:

She works in an office with no work, but high pay.

  • (A) honorary
  • (B) sinecure
  • (C) ex-officio
  • (D) reticent
Correct Answer: (B) sinecure
View Solution

The term "sinecure" refers to a position requiring little or no work but giving the holder status or financial benefit. This perfectly matches the description of an office with no work but high pay.

The other options are incorrect for the following reasons:
- "Honorary" implies a position given as an honor, without the expectation of being paid.

- "Ex-officio" relates to the duties carried out by someone by virtue of their office or position, not related to the absence of work.

- "Reticent" means not revealing one’s thoughts or feelings readily, which is unrelated to the concept described.
Quick Tip: Sinecures are often historical or traditional roles that have evolved to provide remuneration without the corresponding responsibility or workload.


Question 8:


Give one word for the given expression:

Constant effort to achieve something.

  • (A) patience
  • (B) vigour
  • (C) hardship
  • (D) perseverance
Correct Answer: (4) perseverance
View Solution

The term "perseverance" specifically describes the quality of continuing to try to achieve something despite difficulties, which perfectly aligns with the definition of making a constant effort to achieve something.

The other options are incorrect for the following reasons:

- "Patience" refers to the capacity to accept or tolerate delay, problems, or suffering without becoming annoyed or anxious, which does not imply active effort.

- "Vigour" suggests physical strength, energy, or intensity in action, which does not necessarily relate to continuous effort towards a goal.

- "Hardship" implies severe suffering or privation, which is a condition one might endure, rather than the effort itself.
Quick Tip: Perseverance is often the key word when describing sustained efforts to overcome challenges or pursue long-term goals, particularly in contexts requiring determination.


Question 9:


Fill in the blanks:

The ________ of glory lead but to the ________ .

  • (A) ways, frustration
  • (B) paths, grave
  • (C) acts, pride
  • (D) achievements, suffering
Correct Answer: (2) paths, grave
View Solution

The phrase "The paths of glory lead but to the grave" is a famous line from Thomas Gray's poem "Elegy Written in a Country Churchyard." It metaphorically suggests that the pursuit of glory ultimately leads to death, emphasizing the inevitability and universality of mortality.

The other options are incorrect for the following reasons:

- "ways, frustration" and "acts, pride" do not form a common or recognizable phrase within English literature or colloquial use.

- "achievements, suffering" although poetic, does not match the well-known literary reference of Gray's line.
Quick Tip: Familiarity with classic literature and poetry can often provide insight into common references and phrases used in academic and intellectual contexts.


Question 10:


Fill in the blank:

Last Monday _________ a bad day for me as I ____________ the whole of it in hospital bed.

  • (A) was, had spend
  • (B) had been, was spending
  • (C) was, spent
  • (D) were, spent
Correct Answer: (3) was, spent
View Solution

The correct grammatical structure for this sentence requires the simple past tense for both verbs due to the reference to a specific day in the past ("Last Monday"). The verb "was" correctly aligns with the singular subject "Monday," and "spent" is the correct past tense form of "spend."

The other options are incorrect for the following reasons:

- "was, had spend" is incorrect because "had spend" is not grammatically valid; it should be "had spent" if past perfect were needed, which it is not in this context.

- "had been, was spending" might suggest an ongoing condition or action, but is overly complex and unnecessary for the simple past action described.

- "were, spent" is incorrect because "were" does not agree with the singular subject "Monday."
Quick Tip: When dealing with specific points in time in the past, use simple past tense to keep the sentence structure clear and straightforward.


Question 11:


Fill in the blank:

Had Savita worked hard, she __________ have passed the examination.

  • (A) will
  • (B) would
  • (C) shall
  • (D) should
Correct Answer: (2) would
View Solution

The sentence is structured as a third conditional, expressing a hypothetical situation in the past. The correct modal verb to use in this type of conditional sentence is "would," which indicates what would have happened under different circumstances.

The other options are incorrect for the following reasons:

- "will" is used for future intentions or predictions, not suitable for hypothetical past conditions.

- "shall" is used for strong assertions or promises, primarily in a first-person future context, not for past hypotheticals.

- "should" suggests a recommendation or obligation, not a hypothetical outcome.
Quick Tip: Remember, the third conditional structure typically uses "had" + past participle in the if-clause, and "would have" + past participle in the main clause to discuss hypothetical situations that did not happen in the past.


Question 12:


Fill in the blank:

_________ you, please, stop making noise?

  • (A) must
  • (B) need
  • (C) shall
  • (D) will
Correct Answer: (4) will
View Solution

In the context of this sentence, "will" is the appropriate modal verb to use as a polite request. It expresses a polite inquiry or request regarding the willingness of the person to stop making noise.

The other options are incorrect for the following reasons:

- "must" is used to express obligation or necessity, which is too strong and not typically used in polite requests.

- "need" is not conventionally used in this question form to make polite requests.

- "shall" is typically used in formal rules or regulations, or to offer suggestions and not commonly used in the structure of a direct request.
Quick Tip: Using "will" in questions like this is a standard way to make polite requests or inquiries about someone's willingness to do something.


Question 13:


Fill in the blank:

Bhoomi _________ stamps since childhood.

  • (A) was collecting
  • (B) has been collecting
  • (C) were collecting
  • (D) have had been collecting
Correct Answer: (2) has been collecting
View Solution

The correct choice is "has been collecting," which uses the present perfect continuous tense. This tense is used to describe an action that began in the past and continues into the present, especially when emphasizing the duration of the activity ("since childhood").

The other options are incorrect for the following reasons:

- "was collecting" is simple past continuous, used for actions ongoing at a specific time in the past, but not continuing to the present.

- "were collecting" is plural past continuous and does not agree with the singular subject "Bhoomi."

- "have had been collecting" is grammatically incorrect and not a recognized verb tense in English.
Quick Tip: Use the present perfect continuous tense to talk about actions that started in the past and are still continuing, especially when the duration of the action is relevant.


Question 14:


Fill in the blank:

Half of the apples _________ ripe.

  • (A) is
  • (B) are
  • (C) has
  • (D) have
Correct Answer: (2) are
View Solution

The correct choice is "are," which is used here because "half of the apples" refers to a plural subject, indicating a portion of a group of apples. The verb must agree with the plural noun "apples."

The other options are incorrect for the following reasons:

- "is" would only be correct if the subject were singular, but "half of the apples" implies a plural subject.

- "has" and "have" are auxiliary verbs that would require an additional main verb to complete the phrase, which is not provided here. They also do not agree in number or tense with the required form of the verb for this sentence.
Quick Tip: When determining verb agreement, consider whether the noun phrase represents a singular or plural subject, even when fractions or quantities are involved.


Question 15:


Choose the word opposite in meaning to the given word:

PUNGENCY

  • (A) VAPIDITY
  • (B) JAM
  • (C) CRUSE
  • (D) PSORA
Correct Answer: (1) VAPIDITY
View Solution

"Pungency" refers to a strong, sharp taste or smell. The opposite of this would be "vapidity," which means offering nothing that is stimulating or challenging; typically bland or dull.

The other options are incorrect as opposites for the following reasons:

- "JAM" generally refers to a type of condiment or a traffic condition, neither of which pertains to the lack of strong flavor or smell.

- "CRUSE," an old word for a small container for liquids, has no relevance to taste or smell.

- "PSORA" is a medical term related to skin conditions, unrelated to taste or smell.
Quick Tip: Understanding the exact definitions of words helps significantly in identifying correct antonyms, especially when dealing with abstract qualities such as tastes or smells.


Question 16:


Choose the word opposite in meaning to the given word:

'DECIMATE'

  • (A) SCULL
  • (B) WALLOW
  • (C) CONNUBIAL
  • (D) UNITY
Correct Answer: (4) UNITY
View Solution

"Decimate" originally means to destroy a large portion of, often about ten percent, but has come to mean destroy or damage a significant part. The opposite of causing significant destruction would be to create or maintain wholeness or unity. Therefore, "UNITY" is the correct opposite, suggesting harmony and intactness.

The other options are incorrect as opposites for the following reasons:

- "SCULL" refers to a type of small boat or oar, and has no direct antithetical meaning to destruction.

- "WALLOW" means to indulge oneself in a morose or self-pitying way, which does not suggest construction or unity.

- "CONNUBIAL" pertains to marriage or the married state and does not directly contrast with destruction.
Quick Tip: In selecting antonyms, it's crucial to consider the fundamental aspect of the word's meaning—whether it's about forming, maintaining, or dissolving entities or relationships.


Question 17:


Choose the word opposite in meaning to the given word:

'PARSIMONY'

  • (A) LARGESS
  • (B) GERMINAL
  • (C) JOCULAR
  • (D) LIBRETTO
Correct Answer: (1) LARGESS
View Solution

"Parsimony" refers to extreme unwillingness to spend money or use resources; it is synonymous with stinginess. The opposite of this would be "largess," which means generous bestowal of gifts, or the act of giving generously.

The other options are incorrect as opposites for the following reasons:

- "GERMINAL" pertains to something in its early stages of development or something that is productive and generative, which does not directly contrast with stinginess.

- "JOCULAR" relates to humor or playfulness, which is unrelated to the concept of generosity versus stinginess.

- "LIBRETTO" refers to the text of an opera or other long vocal work, which has no connection to financial or resource-based generosity.
Quick Tip: When identifying antonyms, focus on the core concept of the word—financial generosity versus stinginess in this case—to find the most directly opposing term.


Question 18:


Choose the word similar in meaning to the given word:

'VARIEGATION'

  • (A) DIVERSITY
  • (B) DENSITY
  • (C) ATOMISE
  • (D) INSIPID
Correct Answer: (1) DIVERSITY
View Solution

"Variegation" refers to the appearance of differently colored zones in plants, but can more broadly mean variety or diversification. "Diversity" is the closest synonym as it denotes a range of different things, which is similar to variegation in terms of exhibiting or involving variety.

The other options are incorrect for the following reasons:

- "DENSITY" relates to compactness or the number of individuals per unit area, which doesn't connect to variety in terms of differences or variations.

- "ATOMISE" means to break up into very small particles or to divide into many small parts, not directly related to the concept of varied appearances or diversity.

- "INSIPID" describes something lacking flavor or interest, the opposite of what variegation implies in terms of visual or conceptual diversity.
Quick Tip: Understanding the broader context or metaphorical uses of terms like variegation helps in aligning them with the most fitting synonyms, especially when the word is used in non-technical contexts.


Question 19:


Choose the word similar in meaning to the given word:

'TAWDRY'

  • (A) GIMCRACK
  • (B) LACERATE
  • (C) SWAY
  • (D) LEVY
Correct Answer: (1) GIMCRACK
View Solution

"Tawdry" refers to something that is cheap, showy, and of poor quality. "Gimcrack" similarly denotes an object that is showy but cheap and poorly made. Both terms imply a sense of flashy superficiality combined with inferior quality.

The other options are incorrect for the following reasons:

- "LACERATE" means to tear or make deep cuts, which is entirely unrelated to the quality or appearance of objects.

- "SWAY" means to move or influence in a particular direction, not related to physical quality or appearance.

- "LEVY" refers to imposing a tax, fee, or fine, which has no connection to describing the quality or style of objects.
Quick Tip: When finding synonyms, especially for descriptive terms, focus on the adjectives that highlight similar qualities or characteristics in objects, particularly those relating to appearance and quality.


Question 20:


Choose the word similar in meaning to the given word:

'MOGUL'

  • (A) MONACHISM
  • (B) BIGWIG
  • (C) SMART
  • (D) FASHION
Correct Answer: (2) BIGWIG
View Solution

"Mogul" refers to a powerful or influential person, especially in the media or entertainment industries. "Bigwig" is a colloquial term that similarly refers to a person with great influence or authority, often in a particular sphere.

The other options are incorrect for the following reasons:

- "MONACHISM" relates to monastic life or monks, which has no connection to influence or power in a secular or business context.

- "SMART" refers to exhibiting intelligence or cleverness, which does not necessarily imply having power or influence.

- "FASHION" pertains to popular trends, especially in clothing and accessories, and does not relate to a person of influence.
Quick Tip: When choosing synonyms for terms describing societal roles or statuses, it's important to consider the context in which these roles are exercised, especially when they pertain to power or influence.


Question 21:


Spot the error in the given sentence:

My friend is so rich that he is having six houses in Mumbai and four in Pune.

  • (A) A
  • (B) B
  • (C) C
  • (D) D
Correct Answer: (2) B
View Solution

The error in the sentence is in part (B) with the use of "is having." In English, the verb "have" when used to describe possession, should not be in the continuous form. The correct phrasing would be "he has," not "he is having." The use of "is having" generally applies to actions occurring at the moment, such as consuming food (e.g., "He is having lunch").

The other parts of the sentence are correct as follows:

- Part (A): "My friend is so rich that" is a correct lead-in to explain the extent of his wealth.

- Part (C): "six houses in Mumbai and four in Pune" correctly uses numbers and locations.

- Part (D): There is no error in the punctuation or structure in the final segment of the sentence.
Quick Tip: Remember, in English, the verbs that describe states of being, possession, or senses are typically not used in the progressive (continuous) form.


Question 22:

Spot the error in the given sentence:

  • (A) One of the most (B) widely spread (C) bad habit (D) is the use of tobacco. (a) A
    (b) B
    (c) C
    (d) D
Correct Answer: (C) C
View Solution

Step 1: Identifying the Error

The phrase "One of the most" refers to a singular item from a group of plural items. The noun following "one of the" should always be plural.

Step 2: Analyzing the Sentence

- (A) "One of the most" is correct because it properly introduces a comparison.

- (B) "widely spread" is correct and correctly modifies "habit."

- (C) "bad habit" is incorrect. It should be "bad habits" to match the correct grammar structure.

- (D) "is the use of tobacco" is correct and follows proper subject-verb agreement.

Step 3: Choosing the Correct Answer

The error is in (C), where "bad habit" should be "bad habits."



Final Answer: The correct sentence should be: \[ "One of the most widely spread bad habits is the use of tobacco." \] Quick Tip: When using the phrase "one of the," always follow it with a plural noun because it refers to one element from a group.


Question 23:


Spot the error in the given sentence:

  • (A) No method of making other (B)people agree to (C)your view point is as (D)effective as this method.
  • (A) A
  • (B) B
  • (C) C
  • (D) D
Correct Answer: (1) A
View Solution

The error in the sentence is in part (A) where "view point" is mentioned. The correct term should be "viewpoint" as a single word, not two separate words.

The other parts of the sentence are correct as follows:

- Part (B): "of making other people agree to your viewpoint" is grammatically correct and appropriately worded.

- Part (C): "is as effective as this method" correctly completes the comparative structure initiated by "No method...is as effective as."

- Part (D): There is no additional text or punctuation issue.
Quick Tip: It’s important to use the correct form of compound words. "Viewpoint" is often mistakenly written as two separate words, but it should be one to correctly convey the meaning of a perspective or opinion.


Question 24:


Choose the appropriate idiom to fill in the blank:

He invited _________ in the marriage party of his younger brother.

  • (A) all in all
  • (B) alpha and omega
  • (C) all and sundry
  • (D) beck and call
Correct Answer: (3) all and sundry
View Solution

The idiom "all and sundry" means everyone, typically used to suggest that a large, inclusive group of people is involved. This phrase fits perfectly in the context of the sentence, indicating that he invited everyone to the marriage party.

The other options are incorrect for the following reasons:

- "All in all" is generally used to summarize or conclude a discussion, not to indicate inclusivity of a group.

- "Alpha and omega" means the beginning and the end, which does not relate to the idea of inviting people.

- "Beck and call" means to be at someone’s complete command, which is irrelevant to inviting people to a function.
Quick Tip: The idiom "all and sundry" is useful for indicating the inclusion of everyone without exceptions, which is often appropriate in social contexts such as invitations.


Question 25:


Choose the most appropriate option from the following:

'To check by Jowl'

  • (A) to be very near to each other
  • (B) to aspire for impossible thing
  • (C) to make minor changes
  • (D) to be kept waiting
Correct Answer: (1) to be very near to each other
View Solution

The correct phrase and meaning here should be "cheek by jowl," which is a commonly used idiom meaning to be very close together or positioned very closely to one another. It likely has been misprinted or misunderstood as "check by Jowl" in the question.

The other options are incorrect for the following reasons:

- "to aspire for impossible thing" does not relate to proximity or closeness, which the correct idiom implies.

- "to make minor changes" and "to be kept waiting" also do not connect to the meaning of being physically or metaphorically close to someone or something.
Quick Tip: When encountering unfamiliar phrases, consider similar sounding or common idioms that might have been mistaken or misheard, especially in colloquial speech or informal writing.


Question 26:


Choose the most appropriate option for the following idiomatic expression:

The problem of corruption has been hanging fire for the last ten years.

  • (A) to remain unsolved
  • (B) to make all possible efforts
  • (C) to create hindrance
  • (D) to have no strength and support
Correct Answer: (1) to remain unsolved
View Solution

The idiom "hanging fire" means to delay in progression or resolution, remaining unresolved or incomplete. Thus, "to remain unsolved" is the most appropriate option for describing a situation where a problem, like corruption, has not been resolved over a period of time.

The other options are incorrect for the following reasons:

- "to make all possible efforts" does not match the meaning of the idiom, which implies delay or postponement.

- "to create hindrance" and "to have no strength and support" are not directly related to the specific delay or prolonged resolution implied by "hanging fire."
Quick Tip: Understanding idioms like "hanging fire" helps in recognizing descriptions of delays or unresolved situations, commonly used in both business and social contexts.


Question 27:


Choose the correct spelling:

  • (A) MOMENTOUS
  • (B) MOMENTEOUS
  • (C) MOMENTIOUS
  • (D) MOMENTIEOUS
Correct Answer: (1) MOMENTOUS
View Solution

The correct spelling of the word is "momentous," which means of great importance or significance. The other options are misspellings and do not represent any recognized English words. Quick Tip: A useful method to remember the correct spelling of "momentous" is to think of it as relating to a "moment" in time that is significant.


Question 28:


Choose the correct spelling:

  • (A) BEATIFICATION
  • (B) BEAUTIFICATION
  • (C) BEAUTIFASCION
  • (D) BEATIFICASCION
Correct Answer: (2) BEAUTIFICATION
View Solution

The correct spelling is "beautification," which refers to the process of making something visually more beautiful or attractive. The term is derived from the word "beauty." Quick Tip: Remembering the root word "beauty" can help ensure the correct spelling of "beautification," which directly relates to enhancing beauty.


Question 29:


Choose the correct spelling:

  • (A) TOMPOOLARY
  • (B) TOMFOOLARY
  • (C) TOMFOOLERY
  • (D) TOMPOOLERY
Correct Answer: (3) TOMFOOLERY
View Solution

The correct spelling is "tomfoolery," which refers to foolish or silly behavior. The term is derived from the phrase "Tom Fool," historically used to describe a foolish person. Quick Tip: The correct spelling, "tomfoolery," combines "Tom" and "foolery," hinting at its meaning related to foolishness. It's useful to connect the spelling to its etymological roots when in doubt.


Question 30:


What is the duration of a plan?

  • (A) 5 years
  • (B) a period
  • (C) 20 years
  • (D) upto 2017
Correct Answer: (1) 5 years
View Solution

According to the passage, the plans in India were of five years duration and were called five year plans. This indicates that the duration of each plan cycle was typically five years. Quick Tip: When reading passages with specific details about durations or timelines, it's useful to note the exact terms used, such as "five year plans," which directly answer questions about durations or cycles.


Question 31:


What is the purpose of a plan?

  • (A) to figure out the resources of a nation
  • (B) to figure out general goals and objectives
  • (C) to put to use the resources with specific objectives only
  • (D) to put to use the resources keeping in mind goals and objectives to be achieved within a stipulated amount of time
Correct Answer: (4) to put to use the resources keeping in mind goals and objectives to be achieved within a stipulated amount of time
View Solution

The passage outlines that a plan's purpose is to use a nation's resources with both general and specific objectives in mind, to be achieved within a specified timeframe. Option (D) encapsulates this description accurately by mentioning the utilization of resources according to predetermined goals and objectives within a set period. Quick Tip: When considering the purpose of a plan as described in literature or documentation, it's crucial to recognize the emphasis on effective resource utilization paired with time-bound objectives.


Question 32:


Which of the following statements is true?

  • (A) All plans are given equal importance in India.
  • (B) The goals of a plan is to always smooth out any friction.
  • (C) The planners plan the goals and manage the goals effectively.
  • (D) Different goals are being emphasised in different plans in India.
Correct Answer: (4) Different goals are being emphasised in different plans in India.
View Solution

The passage specifically mentions that it is unrealistic to expect all goals of a plan to be given equal importance and that different goals may actually be in conflict. This indicates that different goals are indeed emphasised in different plans in India, making option (D) the correct answer. Quick Tip: When analyzing text for comprehension questions, focus on statements that directly reference the text to find support for the correct answer.


Question 33:


Which of the following statements is false?

  • (A) The basis for the perspective plan is five year plans.
  • (B) Soviet Union spelled out the amount and extend of goods and services successfully.
  • (C) The goals of a plan is to give equal importance to all the plans.
  • (D) The goals of introducing modern technology may produce conflict with the employment plan.
Correct Answer: (3) The goals of a plan is to give equal importance to all the plans.
View Solution

The passage explicitly states that it is unrealistic to expect all goals of a plan to be given equal importance and highlights that different goals may actually be in conflict. This indicates that statement (C) is false as it directly contradicts the information given in the passage. Quick Tip: When assessing statements for truthfulness, cross-reference each statement with the passage to ensure accuracy based on the provided text.


Question 34:


Find from the passage the word similar in meaning to: 'up-to-date'

  • (A) planning
  • (B) modern
  • (C) technology
  • (D) goals
Correct Answer: (2) modern
View Solution

The passage discusses the introduction of modern technology, contrasting it with the goal of increasing employment. The term "modern" in this context aligns with the meaning of "up-to-date," which typically refers to something current or contemporary. Therefore, "modern" is the best fit as it directly relates to being current and new, much like "up-to-date." Quick Tip: The term "up-to-date" generally refers to the latest or most recent advancements, similar to how "modern" is used to describe contemporary or current technologies or methodologies.


Question 35:


What is the objective of the given passage?

  • (A) To encourage a five year plan better.
  • (B) To put into perspective the long-term plan.
  • (C) To justify the failure of plan.
  • (D) To introduce the basics of national planning.
Correct Answer: (4) To introduce the basics of national planning.
View Solution

The passage provides an overview of how national plans are structured and utilized in India, discussing both five-year plans and the long-term 'perspective plan.' The focus is on explaining how these plans function and their objectives, making "to introduce the basics of national planning" the most appropriate objective of the passage. Quick Tip: When identifying the objective of a text, consider what information is being explained or clarified throughout the passage, and which option best summarizes this focus.


Question 36:


Identify the odd one out:


(a) National objectives

(b) Modern technology

(c) Commanding role

(d) Perspective plan

Correct Answer:(A) National objectives
View Solution

Among the given options, National objectives is the odd one out.

- Modern technology, commanding role, and perspective plan are terms that relate to strategic and technological development in planning.
- National objectives, on the other hand, refer to broad goals set by a country that may include social, economic, and political targets, making it conceptually different from the other three.

Thus, National objectives is the odd one out. Quick Tip: When identifying the odd one out, examine whether three options belong to a similar category while one differs. Here, three terms relate to technological and strategic planning, whereas national objectives are a broader concept.


Question 37:


Historians see time as

  • (A) a clock or a calendar.
  • (B) transformation of ideas and beliefs.
  • (C) large segments
  • (D) periods that possess shared characteristics.
Correct Answer: (4) periods that possess shared characteristics.
View Solution

The passage states that historians view time not just as a clock or a calendar but also consider it as a reflection of changes in social and economic organization, encapsulated in large segments or periods with shared characteristics. Therefore, the correct answer is option (D), as it aligns with how historians perceive time through periods that share defining traits. Quick Tip: When interpreting how professionals in a field view abstract concepts like time, it's important to consider their broader, more analytical perspectives that go beyond simple, everyday meanings.


Question 38:


British historians divided the history of India into -

  • (A) periods based on social and economic organisation.
  • (B) periods based on religion of people.
  • (C) periods based on important historical changes.
  • (D) periods based on religion of rulers.
Correct Answer: (4) periods based on religion of rulers.
View Solution

According to the passage, British historians divided the history of India into three periods named "Hindu", "Muslim", and "British". This division was primarily based on the religion of the rulers during those periods, not on the religion of the people or other social and economic factors. Therefore, option (D) accurately reflects this focus on the religion of rulers as the basis for historical periodization. Quick Tip: When interpreting historical divisions or classifications, it's essential to recognize the criteria used by historians, which may reflect contemporary perspectives and biases of the time.


Question 39:


Choose the correct statement - The division made by the British historians

  • (A) was based on shared characteristics of people.
  • (B) reflected changes in social and economic organisation.
  • (C) was based on significant developments
  • (D) ignored the rich diversity of the subcontinent.
Correct Answer: (4) ignored the rich diversity of the subcontinent.
View Solution

The passage explicitly mentions that the British historians' division of Indian history into "Hindu", "Muslim", and "British" periods based solely on the religion of rulers ignored other significant developments in the economy, society, or culture. This approach also overlooked the rich diversity of the subcontinent, making option (D) the correct statement. Quick Tip: When evaluating historical analyses, it's crucial to consider what aspects of history might have been overlooked or simplified due to the biases or perspectives of the historians.


Question 40:


Most historians today divide the history of India on the basis of -

  • (A) social and religious factors taken together.
  • (B) economic and social factors taken together.
  • (C) 'Hindu', 'Muslim' and 'British' rulers.
  • (D) large segments of time.
Correct Answer: (2) economic and social factors taken together.
View Solution

The passage notes that few historians follow the old method of dividing Indian history based on the religion of rulers, which was predominant in the past. Instead, contemporary historians tend to look at economic and social factors to characterize the major elements of different moments of the past, making option (B) the most accurate reflection of current historical analysis practices. Quick Tip: It's important to recognize shifts in historiographical methods over time, as they often reflect broader changes in societal values and academic understanding.


Question 41:


The meaning of the word 'persistence' as per the context is -

  • (A) credulity
  • (B) disapprobation
  • (C) continuity
  • (D) disappearance
Correct Answer: (3) continuity
View Solution

In the context of the passage, 'persistence' refers to the enduring nature of ideas and beliefs through time. This meaning aligns with the concept of 'continuity', which denotes the unbroken and consistent existence or operation of something over a period of time. Thus, 'continuity' is the correct interpretation of 'persistence' in this context. Quick Tip: When interpreting words like 'persistence', consider how they are used in relation to other concepts in the text, such as changes or transformations, to understand their nuanced meanings.


Question 42:


The British historians carried out the division of history of India -

  • (A) around 1750 AD.
  • (B) around 1850 AD.
  • (C) around 1950 AD.
  • (D) around 1900 AD.
Correct Answer: (2) around 1850 AD.
View Solution

The passage indicates that in the middle of the nineteenth century, British historians divided the history of India into three periods: "Hindu", "Muslim" and "British". Considering that the mid-nineteenth century refers to around the 1850s, option (B) is the most accurate. Quick Tip: When dealing with historical dates, it's important to consider the context provided by the source material. The mid-nineteenth century typically refers to the 1850s, which helps in precisely dating such events.


Question 43:


Which of the following views does the author fully support in the passage?

  • (A) Nothing is permanent as change.
  • (B) Change is always rapid.
  • (C) More money spent on innovation leads to more rapid change.
  • (D) Over decades structural changes have been incremental.
Correct Answer: (1) Nothing is permanent as change.
View Solution

The passage strongly supports the view that "nothing is as permanent as change." This idea is expressed directly in the text, emphasizing that change is a constant, even if not always rapid or drastic. The author mentions that while people may fear change, it is an inevitable and ongoing part of life, highlighting the permanence of change over time. Quick Tip: Understanding the themes of permanence and transformation in texts helps in grasping deeper philosophical views expressed by authors, especially in discussions about societal or technological changes.


Question 44:


Any significant cultural or social change -

  • (A) depends upon technological advancement.
  • (B) tends to move in the direction of permanence.
  • (C) is countered by the pillars of tribalism.
  • (D) is carried out for a good reason.
Correct Answer: (3) is countered by the pillars of tribalism.
View Solution

The passage clearly supports the idea that any significant cultural or social change is often resisted or countered by the pillars of tribalism. William Ury, discussed in the passage, explains that tribalism serves to protect people from the fears associated with rapid change, thereby countering significant cultural or social changes. Quick Tip: In discussions about societal dynamics, it's crucial to understand how traditional social structures, like tribalism, can serve as mechanisms to resist or slow down change, providing a sense of security and continuity.


Question 45:


According to the IBM CEO -

  • (A) change is scary.
  • (B) the competitors will force us to change.
  • (C) our fathers and grandfathers lived in a period of little change.
  • (D) we have not changed the structure speedily.
Correct Answer: (3) our fathers and grandfathers lived in a period of little change.
View Solution

The passage includes a reflection from an IBM CEO stating, "We only restructure for a good reason, and if we haven't restructured in a while, that's a good reason." It also mentions asking fathers and grandfathers if they lived in a period of little change, suggesting that the perception may be of little change, even though changes continue to occur. This aligns with option (C), which emphasizes the perception that older generations experienced less change compared to the rapid changes observed today. Quick Tip: It's insightful to consider how perceptions of change can vary significantly between generations, often influenced by the technological and social dynamics of their respective times.


Question 46:


Anthropology is the study of

  • (A) art of negotiation.
  • (B) human origins, customs and beliefs.
  • (C) human congregations in a given time period.
  • (D) art of growing a garden.
Correct Answer: (2) human origins, customs and beliefs.
View Solution

Anthropology is a field of study focused on human societies, their developments, and the various aspects that define cultural and social identities, including origins, customs, and beliefs. This aligns directly with option (B), making it the accurate description of anthropology. Options (A), (C), and (D) do not represent the primary focus of anthropological studies. Quick Tip: Anthropology encompasses a wide range of human aspects, emphasizing the importance of understanding past and present human behaviors and cultural practices.


Question 47:


According to Prof. William Ury -

  • (A) change happens too fast.
  • (B) the traditional and familiar will assume greater importance.
  • (C) technology is a pillar of society.
  • (D) tribalism protects people from their fear of rapid change.
Correct Answer: (4) tribalism protects people from their fear of rapid change.
View Solution

The passage discusses William Ury's views on how tribalism serves as a defense mechanism against the fear of rapid change. According to Ury, tribalism's role is to provide a sense of security and continuity, thus countering significant cultural or social changes that might otherwise cause insecurity and fear among people. Quick Tip: Understanding the psychological and social functions of traditional structures like tribalism can offer insights into why societies resist or adapt to change.


Question 48:


The meaning of the word 'pithy' is -

  • (A) expressed in a clear, direct way.
  • (B) a phrase or idea that has been used many times.
  • (C) an important idea.
  • (D) ambiguous and unclear idea.
Correct Answer: (1) expressed in a clear, direct way.
View Solution

The word 'pithy' refers to language or speech that is concise and forcefully expressive. It typically means something expressed in a clear, direct, and often witty manner, rather than an idea that has been used many times or is ambiguous. Thus, option (A) best captures the essence of being 'pithy'. Quick Tip: Pithy statements are valued for their ability to convey significant meanings or truths in a few well-chosen words, often using wit or sharpness.


Question 49:


The amount of money MobiKwik plans to raise through issue of shares is -

  • (A) Rs. 700 crores
  • (B) Rs. 1900 crores
  • (C) Rs. 140 crores
  • (D) 50%\ lower than 2021
Correct Answer: (1) Rs. 700 crores
View Solution

According to the passage, One MobiKwik Systems filed a fresh draft red herring prospectus with Sebi to raise Rs. 700 crore through a fresh issue of shares. This directly corresponds with option (A), confirming that the planned amount for the current IPO is Rs. 700 crores. Quick Tip: When analyzing IPO announcements, it's essential to distinguish between the amount to be raised through the public offering and any other financial arrangements or historical figures presented.


Question 50:


MobiKwik stalled its first IPO plans because of

  • (A) uncertain markets
  • (B) violent markets
  • (C) dull markets
  • (D) jerky markets
Correct Answer: (1) uncertain markets
View Solution

The passage specifies that MobiKwik stalled its IPO plans on the back of choppy markets. The term "choppy markets" can be best interpreted as "uncertain markets," which captures the volatility and unpredictability that led to the stalling of the IPO. This matches option (A), making it the correct choice. Quick Tip: In financial contexts, "choppy" often describes markets that are characterized by rapid short-term fluctuations, indicative of uncertainty and instability.


Question 51:


The money collected through IPO will be used by MobiKwik -

  • (A) to sustain the company.
  • (B) to raise its funds ahead of Paytm.
  • (C) to fund its growth in the market and invest in data and technology tools.
  • (D) to fund the growth of its financial services businesses and invest in data and technology tools.
Correct Answer: (4) to fund the growth of its financial services businesses and invest in data and technology tools.
View Solution

The passage states that MobiKwik will use the IPO proceeds to fund growth of its financial services and payment services businesses and make investments in data and technology tools like AI \& ML. This corresponds directly with option (D), which specifically mentions the utilization of funds for the expansion of financial services and investments in technology. Quick Tip: When evaluating how companies plan to use funds raised from IPOs, it's crucial to look for specific areas of investment that align with their strategic growth plans and technological advancement.


Question 52:


Give one word for the following phrase - 'goods that are bought or sold'

  • (A) prospectus
  • (B) issue
  • (C) bourses
  • (D) merchandise
Correct Answer: (4) merchandise
View Solution

The correct term for goods that are bought or sold is "merchandise." This word directly refers to products or goods that are traded or sold in various markets, making option (D) the appropriate choice for the given phrase. Quick Tip: "Merchandise" typically encompasses all categories of goods that can be bought or sold, making it a broad term used in commerce.


Question 53:


Choose the correct synonym for the word 'dismal'

  • (A) promising
  • (B) thundering
  • (C) depressing
  • (D) considerable
Correct Answer: (3) depressing
View Solution

The word 'dismal' commonly refers to something that is dreary, causing a mood of gloom or depression. Thus, the synonym that best matches 'dismal' is 'depressing', corresponding to option (C). Quick Tip: The word 'dismal' often connotes negative emotional states, which makes 'depressing' a fitting synonym, especially in contexts relating to mood or atmosphere.


Question 54:


ABD is filing its IPO for Rs. 2,000 crore -

  • (A) to expand its business
  • (B) to retire part of its debt
  • (C) to payoff its debt and expand its business
  • (D) to foray into premium segment and compete with global players.
Correct Answer: (3) to payoff its debt and expand its business
View Solution

The passage indicates that the IPO will be a combination of a fresh fund-raising and an offer for sale by some promoters, with the proceeds used partly to retire debt and partly to expand the business, including launching new brands. Therefore, option (C) most accurately reflects the purposes outlined for the IPO, covering both retiring part of the debt and expanding the business. Quick Tip: In IPO contexts, it's crucial to understand the specific use of raised funds as it provides insights into the company's strategic financial planning and future growth areas.


Question 55:


Deduce from the passage-Officer's choice is \underline{\hspace{3cm as compared to Sterling Reserve.

  • (A) Costlier
  • (B) Cheaper
  • (C) Dearer
  • (D) Equally priced
Correct Answer: (2) Cheaper
View Solution

The passage mentions that ABD forayed into the premium segment with Sterling Reserve in 2017, implying that Sterling Reserve is positioned as a premium product. Given that Officer's Choice has been described as the flagship brand and not specifically mentioned within the premium context like Sterling Reserve, it can be deduced that Officer's Choice is likely cheaper compared to Sterling Reserve, which is targeted at a higher-end market segment. Quick Tip: When assessing product pricing strategies, understanding the market positioning—such as 'premium'—can provide key insights into relative pricing among a company's different brands.


Question 56:


The earliest company to file its IPO according to the passage is -

  • (A) Allied Blenders \& Distillers
  • (B) Diageo
  • (C) Pernod Ricard
  • (D) Globus Spirits
Correct Answer: (4) Globus Spirits
View Solution

The passage mentions that Globus Spirits was the last such company to go public in 2009, prior to the planned IPO of ABD. This indicates that among the options listed, Globus Spirits filed its IPO earliest relative to the context provided in the passage. Quick Tip: Historical IPO dates for companies within the same sector can provide valuable benchmarks for analyzing industry trends and investment cycles.


Question 57:


Give one word for the following group of words - 'The most important product, service, building, etc. that an organisation owns or produces':

  • (A) domestic
  • (B) valuation
  • (C) brand
  • (D) flagship
Correct Answer: (4) flagship
View Solution

The term "flagship" is commonly used to refer to the most important product, service, or entity within a company's portfolio that represents its brand and market presence. This term is the perfect match for describing a principal product or service that symbolizes the organization's leading offerings in the market. Quick Tip: The term "flagship" often indicates the primary or most notable product, especially in contexts where companies have multiple offerings but are predominantly known for one.


Question 58:


Choose the most appropriate word to fill in the blank:

I had to pay extra amount for a \underline{\hspace{3cm car number.

  • (A) domestic
  • (B) public
  • (C) popular
  • (D) premium
Correct Answer: (4) premium
View Solution

The term "premium" is typically used to describe something that requires an additional cost due to its superior quality, desirability, or exclusivity. In the context of paying extra for a car number, "premium" is the most suitable word, implying that the car number is special or sought-after enough to warrant additional charges. Quick Tip: "Premium" can often relate to services or goods that are considered above standard, which often require an extra cost reflecting their higher perceived value.


Question 59:


The difference in the Audi sales in the years 2023 and 2020 is -

  • (A) approximately 5500 units
  • (B) less than 6000 units
  • (C) more than 6000 units
  • (D) 89% over 2022
Correct Answer: (C) more than 6000 units
View Solution

From the passage, Audi's sales in 2020 were 1,639 units and in 2023 were 7,931 units. The difference in sales between these two years is calculated as follows: \[ 7,931 - 1,639 = 6,292 \, units \]
Thus, the difference is more than 6000 units. Quick Tip: When calculating differences or changes between numerical data across years, always subtract the earlier year's figure from the later year's figure to find the net change.


Question 60:


The total number of differences in the given sentence, from the passage are -

  • (A) 3
  • (B) 4
  • (C) 5
  • (D) 6
Correct Answer: (B) 4
View Solution

Analyzing the provided sentence: "The surge came even when their close rivals and compatriots Mercedes-Benz and BMW have strong petrol line-ups in their portfolio." Comparing with the context in the passage, key differences are:
- The mention of "compatriots" which is not specified in the passage.
- Emphasis on "strong petrol line-ups," whereas the passage discusses general strategies.
- The timing and causality indicated by "even when" could be interpreted differently.
- The specific mention of "portfolio" is a detailed addition not covered directly in the passage. Quick Tip: When asked to find differences from the passage, focus on differences in terminology, emphasis, and any added or omitted details that change the understanding of the statement.


Question 61:


Audi achieved the highest sale in India in -

  • (A) 2011
  • (B) 2020
  • (C) 2022
  • (D) 2023
Correct Answer: (A) 2011
View Solution

According to the passage, Audi's sales numbers were highest in the year 2011, reaching 11,192 units, marking the peak of their sales performance in India up to the present context of 2023. Quick Tip: When comparing data points across different years, it is essential to reference the specific figures or descriptions provided in the text to accurately identify peak or significant trends.


Question 62:


The parent company of 'Audi' is -

  • (A) Mercedes-Benz
  • (B) BMW
  • (C) Volkswagen
  • (D) Audi India
Correct Answer: (C) Volkswagen
View Solution

According to the passage, Audi is part of the Volkswagen group's ecosystem, which includes other brands such as Skoda, VW, Lamborghini, and Porsche. Quick Tip: Understanding corporate hierarchies and brand affiliations can be crucial for correctly identifying relationships and ownership structures within large multinational corporations.


Question 63:


Choose the word that means the same as 'a person who is a citizen of the same country as somebody else'.

  • (A) rival
  • (B) compatriot
  • (C) local
  • (D) giant
Correct Answer: (B) compatriot
View Solution

The term "compatriot" refers to someone who is from the same country as someone else, reflecting a shared nationality. Quick Tip: Understanding terms related to nationality and citizenship can enhance comprehension of context in discussions about social and political dynamics.


Question 64:

Choose the incorrect statement:

  • (A) The Supreme Court declined to interfere in Sebi's investigation.
  • (B) At the close of the day's session, not all of the 10 Adani group stocks were in the green.
  • (C) The group's total market cap also crossed the Rs. 15-lakh-crore mark in Wednesday's session.
  • (D) Among the group's stocks, Adani Power is the biggest gainer.
Correct Answer: (B) At the close of the day's session, not all of the 10 Adani group stocks were in the green.
View Solution



Step 1: Identifying the incorrect statement by analyzing the passage content.
According to the passage, all 10 Adani Group stocks were in the green at the close of the day's session, which contradicts option (B). Quick Tip: It is crucial to carefully read the wording of each statement in the options when answering questions about identifying incorrect statements to ensure accuracy in understanding what the passage actually says.


Question 65:

Adani group has had the all time high of:

  • (A) 15.1 lakh-crore
  • (B) 19.2 lakh-crore
  • (C) 43 lakh-crore
  • (D) 68 lakh-crore
Correct Answer: (B) 19.2 lakh-crore
View Solution



Step 1: Identifying the all-time high market cap from the passage.

The passage indicates that the all-time high total market cap of Adani Group was Rs 19.2 lakh crore on January 24, 2023.

% Extra vertical space before the quick tip box Quick Tip: Always verify the figures given in the options with the data provided in the passage to avoid errors, especially with financial figures which can be similar.


Question 66:

Hindenburg report alleged that Adani Group:

  • (A) interfered in Sebi's investigation before Supreme Court's judgement.
  • (B) Crossed the Rs. 15 lakh-crore mark in Wednesday's session.
  • (C) had cooked account-books and round tripped own funds to jack down stock prices.
  • (D) had indulged in serious corporate malfeasance.
  • (D) had indulged in serious corporate malfeasance.
Correct Answer:(D) had indulged in serious corporate malfeasance.
View Solution



Step 1: Understanding the allegations by Hindenburg.

The Hindenburg report specifically accused Adani Group of serious corporate malfeasance, including cooking books of accounts and round-tripping of funds, intended to manipulate stock prices.

% Extra vertical space before the quick tip box Quick Tip: Critical reading of reports and understanding the specific allegations is essential for comprehending the impacts on corporate reputation and stock performance.


Question 67:

Choose one word for the given group of words - the curved path of something that has been fired, hit or thrown into the air:

  • (A) rallied
  • (B) transmission
  • (C) flagship
  • (D) trajectory
  • (D) trajectory
Correct Answer:(D) trajectory
View Solution



Step 1: Define the term.

The term "trajectory" specifically refers to the path that an object follows as it moves through space, particularly when it is launched, shot, or otherwise propelled.

% Extra vertical space before the quick tip box Quick Tip: In physics, understanding trajectories helps predict where any object launched into the air will land, based on its initial velocity and the forces acting upon it, such as gravity and air resistance.


Question 68:

The word closest in meaning to 'malfeasance' is:

  • (A) malintentions
  • (B) malpractice
  • (C) manifestation
  • (D) deficiency
Correct Answer:(B) malpractice
View Solution



Step 1: Understanding 'malfeasance'.

The term 'malfeasance' refers to the performance of an act in an official capacity that is illegal or wrongful, particularly one that is deliberately harmful.

% Extra vertical space before the quick tip box Quick Tip: When dealing with synonyms, focus on the connotations and the context in which words are typically used. 'Malpractice' aligns closely because it denotes professional misconduct or failure to meet standard practices.


Question 69:


The Renaissance art is characterized by -
(A) splendour
(B) shading and perspective
(C) patterns and analogies
(D) harmonies of geomet

Correct Answer: (D) harmonies of geometry
View Solution




The passage explicitly mentions how Leonardo studied optics, mastered shading and perspective, and aimed to understand the harmony of geometry in art. This is confirmed with his work on depicting objects as they appear to the eye, a key trait of Renaissance art. Hence, the correct answer is (D) harmonies of geometry. Quick Tip: In Renaissance art, the focus was on perfecting the use of geometry, light, and perspective to create more realistic and harmoniously proportioned representations.


Question 70:


Choose the most appropriate statement that describes Vinci's vision -

(A) He had a limited capacity to marvel at the world.
(B) He was illegitimate and left-handed.
(C) He apprenticed at a workshop, learning to sculpt bodies in motion.
(D) He saw human bodies as microcosms of the universe.

Correct Answer: (D) He saw human bodies as microcosms of the universe.
View Solution




In the passage, it is mentioned that Leonardo was driven by curiosity and sought to find analogies between the human body and the world. His vision of human bodies as microcosms of the universe reflects this idea, making option (D) the most appropriate choice. Quick Tip: Leonardo's curiosity led him to believe in the deep interconnectedness between the human body and the universe, symbolizing his broader scientific and artistic approach.


Question 71:


The author of this passage considers Leonardo da Vinci a 'more fanciful mortal' rather than a genius because -

(A) his accomplishments sprang from his capacity to marvel at the world.
(B) he was not formally schooled but was 'a disciple of experience'.
(C) he was impelled by scientific humanism unbothered by religious dogma.
(D) he had unfinished masterpieces, impractical fantasies and flying machines that never flew.

Correct Answer: (D) he had unfinished masterpieces, impractical fantasies and flying machines that never flew.
View Solution




The passage describes Leonardo da Vinci as having had unfinished works, impractical ideas, and concepts that never materialized, such as flying machines. This is why the author refers to him as a "more fanciful mortal," making option (D) the correct answer. Quick Tip: Da Vinci's greatness lies not only in his accomplishments but in his constant experimentation, often leading to fantastical and unfulfilled ideas, showcasing his creative and imaginative approach.


Question 72:


Give one word for the following group of words - 'Mysterious and difficult to understand'

(A) marvel
(B) weird
(C) enigmatic

Correct Answer: (C) enigmatic
View Solution




The word "enigmatic" accurately captures the meaning of being mysterious and difficult to understand, making it the correct choice in this context. The other options do not convey the same level of mystery or difficulty in understanding. Quick Tip: When looking for synonyms, focus on the precise meaning of the word. "Enigmatic" specifically refers to something that is puzzling and hard to understand.


Question 73:


The closest synonym of the word 'ugly' is -

(A) splendoured
(B) impelled
(C) weird
(D) grotesque

Correct Answer: (D) grotesque
View Solution




The word "grotesque" closely matches the meaning of "ugly," as both refer to something that is unpleasant or distorted in appearance. The other options do not share this meaning. Quick Tip: For choosing synonyms, consider the context in which the word is used. "Grotesque" is commonly used to describe something unpleasantly ugly or distorted.


Question 74:


The reason for Gautam Adani regaining his position of being the wealthiest person in Asia is -


(a) A roller-coaster ride on the wealth rankings last year.

(b) The Supreme Court saying that no new probes were needed into the allegations against the Adani Group.

(c) The Bloomberg Billionaires Index shows him ahead of Mukesh Ambani.

(d) That it has been able to woo back the investors and lenders.

Correct Answer:(B) The Supreme Court saying that no new probes were needed into the allegations against the Adani Group.
View Solution

Gautam Adani's return to being the wealthiest person in Asia is mainly attributed to the Supreme Court's decision that no new investigations were required into allegations against the Adani Group.

While other factors like the roller-coaster wealth rankings and the Bloomberg Billionaires Index were involved, the ruling provided a significant boost to investor confidence, leading to a rise in Adani’s net worth. Quick Tip: When determining the most relevant answer, focus on the primary cause rather than secondary effects. Here, the Supreme Court’s decision played a crucial role in driving market co


Question 75:


The net worth of Adani Group at the time of writing this article is -


(a) 97.6 billion

(b) 150 billion

(c) 13.3 billion

(d) 118.9 billion

Correct Answer:(A) 97.6 billion
View Solution

The passage explicitly states that Gautam Adani's net worth at the time of writing was
(97.6 billion.

While other figures might relate to different time periods or estimations, the correct answer is the exact figure mentioned in the article. Quick Tip: For questions involving numerical values, always refer directly to the figures stated in the passage rather than assuming or estimating from trends.


Question 76:


Adani Group has committed to -


(a) Regain its market share.

(b) Regain its investors and lenders.

(c) Repay its debts and assuage regulatory concerns.

(d) Go ahead with green transition across its businesses.

Correct Answer:(D) Go ahead with green transition across its businesses.
View Solution

The passage highlights that the Adani Group has committed to a green transition across its businesses. This means the company is focusing on renewable energy, sustainability, and eco-friendly initiatives as part of its long-term strategy.

While repaying debts and addressing regulatory concerns were important steps for stabilizing the group, the primary commitment is towards a green transition, making option (D) the correct answer. Quick Tip: When analyzing corporate commitments, focus on long-term strategic goals rather than short-term actions. In this case, Adani Group's transition to green energy is a forward-looking initiative, making it the most relevant answer.


Question 77:


Choose the appropriate word to fill in the blanks -

The Cabinet ________ behind the Prime Minister.

  • (A) needed
  • (B) started off
  • (C) rallied
  • (D) stoked
Correct Answer: (C) rallied
View Solution



The sentence implies that the Cabinet strongly supported the Prime Minister, which is why "rallied" is the correct word to fill in the blank. The word "rallied" is commonly used in contexts where there is a gathering of support or encouragement, especially in political or motivational settings.

% Space above quicktip box Quick Tip: In context-based questions like this, words with stronger, more active connotations that fit the tone of the sentence are often the best choice. "Rallied" indicates a clear, collective action behind the Prime Minister.


Question 78:


Choose one word for the given group of words – 'to make an unpleasant feeling less severe'

  • (A) probe
  • (B) bombshell
  • (C) assuage
  • (D) reprieve
Correct Answer: (C) assuage
View Solution



The word "assuage" means to make an unpleasant feeling less severe or to relieve. This makes it the most suitable word to fill in the blank. "Probe" refers to an investigation, "bombshell" refers to a sudden and unexpected event, and "reprieve" means to delay or cancel a punishment, none of which fit the context.

% Space above quicktip box Quick Tip: When looking for a word that reduces the intensity of something unpleasant, "assuage" is commonly used to imply relief or lessening of a negative feeling.


Question 79:


The event starting from February 1 is called:


(A) Auto Expo

(B) Bharat Mobility Expo

(C) Mobility Show

(D) Noida Expo

Correct Answer: (B) Bharat Mobility Expo
View Solution

The passage mentions that the event starting from February 1 is specifically called Bharat Mobility Expo, as introduced by the government with support from various agencies. This event is designed to bring together multiple sectors such as car makers, components, and electric vehicles under one platform. Hence, the correct answer is option (B).

% Adds space above the quicktip box Quick Tip: Always refer to specific mentions in the passage, like event names, dates, and organizations, which help answer questions accurately.


Question 80:


The new name for Pragati Maidan is:


(A) Bharat Mandapam

(B) Yashobhoomi

(C) Expo Centre

(D) Noida Expo centre

Correct Answer: (A) Bharat Mandapam
View Solution

According to the passage, the new name for Pragati Maidan has been mentioned as Bharat Mandapam. This name change is part of the government's push to revitalize the venue and include it as part of the newly established Bharat Mobility Expo. Hence, the correct answer is option (A).

% Adds space above the quicktip box Quick Tip: When dealing with events or locations mentioned in passages, always refer to specific terminologies used within the passage to answer correctly.


Question 81:


The 'M' in MICE stands for:


(A) Mobility

(B) Manufacturers

(C) Mercedez

(D) Meetings

Correct Answer: (D) Meetings
View Solution

The acronym MICE stands for Meetings, Incentives, Conferences, and Exhibitions. As mentioned in the passage, the term "MICE" refers to an industry for organizing conferences and exhibitions. Therefore, the 'M' in MICE represents 'Meetings'. Hence, the correct answer is option (D).

% Adds space above the quicktip box Quick Tip: In industry-specific abbreviations, make sure to refer to the context within the passage to correctly identify the meaning of acronyms.


Question 82:


The total number of differences in the given sentence, from the passage are -

For a moment, the society of Indian Automobiles manufacturers or Siam, the Auto Expo was sticking to their plan.


(A) 3

(B) 4

(C) 5

(D) 6

Correct Answer: (B) 4
View Solution

In the given sentence, "For a moment, the society of Indian Automobiles manufacturers or Siam, the Auto Expo was sticking to their plan," there are 4 differences:

- The word "Automobiles" should be "Automobile."

- The term "Siam" seems to be a misplaced word.

- The sentence uses "the" before "Auto Expo" which does not align with the usual structure.

- The conjunction "or" is not required in this sentence.
Thus, the number of differences is 4.

% Adds space above the quicktip box Quick Tip: Carefully read the passage and sentence to identify word discrepancies, subject-verb agreement issues, and unnecessary words.


Question 83:


Choose the word that means the same as:
'Saying what you think openly and honestly'


(A) talking

(B) pitching

(C) conference

(D) candid

Correct Answer: (D) candid
View Solution

The word 'candid' refers to speaking honestly and openly, without hiding anything or being deceitful. The phrase 'Saying what you think openly and honestly' is a description of being candid. Hence, the correct answer is option (D).

% Adds space above the quicktip box Quick Tip: When choosing the correct word for definitions, make sure to closely align with the context or the meaning provided in the passage.


Question 84:


A and B entered into a business partnership with Rs. 50,000 and Rs. 60,000, respectively. C joined them after \( X \) months by contributing Rs. 70,000, and B left \( X \) months before the end of the year. If they shared the profit in the ratio 20:18:21, then the value of \( X \) is:


(a) 3

(b) 6

(c) 8

(d) 9

Correct Answer: \textbf{(A) 3}
View Solution

Step 1: Define Capital Contributions and Duration

- A invested Rs. 50,000 for 12 months.
- B invested Rs. 60,000 but left \( X \) months before the end of the year, meaning B invested for \( (12 - X) \) months.
- C joined \( X \) months after the beginning of the year with Rs. 70,000, meaning C invested for \( (12 - X) \) months.

Step 2: Calculate the Capital-Time Products

Each partner’s profit share is proportional to Capital × Time, so we set up the equations:
\[ A’s share = 50000 \times 12 \]
\[ B’s share = 60000 \times (12 - X) \]
\[ C’s share = 70000 \times (12 - X) \]

Step 3: Set Up the Ratio Equations

Since the profit-sharing ratio is 20:18:21, we write:
\[ \frac{50000 \times 12}{60000 \times (12 - X)} = \frac{20}{18} \]
\[ \frac{60000 \times (12 - X)}{70000 \times (12 - X)} = \frac{18}{21} \]

Step 4: Solve for \( X \)

1. From the first equation:
\[ \frac{50000 \times 12}{60000 \times (12 - X)} = \frac{20}{18} \]
\[ \frac{50000 \times 12}{60000} = \frac{20}{18} (12 - X) \]

Solving, we get \( X = 3 \).

Final Answer: (A) 3 Quick Tip: When solving partnership problems, remember that profit is shared in proportion to the product of capital and time. Always set up the correct ratio equations based on this principle.


Question 85:


After allowing a discount of 10% on the marked price, a trader still makes a profit of 17%. By what percentage is the marked price above the cost price?

(A) 20%

(B) 30%

(C) 28%

(D) 27%

Correct Answer:(B) 30%
View Solution

Let the cost price (CP) = 100 units.
Let the marked price (MP) = x units.

After allowing a 10% discount, the selling price (SP) becomes: \[ SP = 0.9 \times MP \]

Given that the trader makes a 17% profit, we use: \[ SP = 1.17 \times CP \]

Substituting values: \[ 0.9 \times MP = 1.17 \times 100 \]
\[ MP = \frac{1.17 \times 100}{0.9} = 130 \]

Percentage increase in marked price over cost price:
\[ \frac{MP - CP}{CP} \times 100 = \frac{130 - 100}{100} \times 100 = 30% \]

Thus, the marked price is 30% above the cost price, making the correct option (B) 30%. Quick Tip: For questions involving profit, discount, and marked price, always establish relationships using the formulas: \[ SP = MP - Discount \] \[ SP = CP + Profit \] Then, solve for the unknown variable.


Question 86:


Sugar syrup A contains 40% sugar and Sugar syrup B contains 80% sugar. In what ratio Syrup A and Syrup B be mixed to obtain a syrup with 65% sugar?


(A) 2:3

(B) 3:4

(C) 3:5

(D) 4:5

Correct Answer: (C) 3:5
View Solution

Step 1: Let the quantity of syrup A and syrup B be \( x \) and \( y \) respectively.

Step 2: The total sugar content from both syrups should be 65% of the total quantity \( x + y \).

Step 3: From the given information:
- Syrup A contains 40% sugar: The amount of sugar from syrup A is \( 0.40x \).
- Syrup B contains 80% sugar: The amount of sugar from syrup B is \( 0.80y \).

Step 4: The total amount of sugar is \( 0.65(x + y) \).
\[ 0.40x + 0.80y = 0.65(x + y) \]

Step 5: Simplifying the equation: \[ 0.40x + 0.80y = 0.65x + 0.65y \]
\[ 0.40x - 0.65x = 0.65y - 0.80y \]
\[ -0.25x = -0.15y \]

Step 6: Solving for the ratio of \( x \) to \( y \):
\[ \frac{x}{y} = \frac{0.15}{0.25} = \frac{3}{5} \]

Thus, the required ratio is \( 3:5 \).

% Adds space above the quicktip box Quick Tip: To solve mixture problems, use the concept of the weighted average sugar content and set up an equation based on the total amount of sugar.


Question 87:


A man and a woman can do a piece of work in 7 days and 8 days, respectively. If both finish the work with the help of a boy in 3 days and get payment of Rs. 1,400, then the share of the boy is:


(A) Rs. 275

(B) Rs. 525

(C) Rs. 600

(D) Rs. 250

Correct Answer: (A) Rs. 275
View Solution

Step 1: Let the amount of work done per day by the man, woman, and boy be denoted as \( M \), \( W \), and \( B \), respectively.
\[ M = \frac{1}{7}, \quad W = \frac{1}{8}, \quad B = \frac{1}{x} \]
where \( x \) is the number of days the boy takes to complete the work alone.

Step 2: Since together they complete the work in 3 days, the equation becomes: \[ 3 \left( \frac{1}{7} + \frac{1}{8} + \frac{1}{x} \right) = 1 \] \[ \frac{1}{7} + \frac{1}{8} + \frac{1}{x} = \frac{1}{3} \]

Step 3: Solving for \( x \): \[ \frac{1}{7} + \frac{1}{8} = \frac{15}{56} \] \[ \frac{15}{56} + \frac{1}{x} = \frac{1}{3} \] \[ \frac{1}{x} = \frac{1}{3} - \frac{15}{56} = \frac{56 - 45}{168} = \frac{11}{168} \] \[ x = \frac{168}{11} = 15.27 (approximately 15 days) \]

Step 4: Now, the work done by the boy in 3 days is: \[ Boy's work in 3 days = 3 \times \frac{1}{15.27} \approx \frac{1}{5.09} \]

Step 5: The payment is Rs. 1,400. The boy's share is: \[ Boy's share = \frac{1}{5.09} \times 1400 \approx 275 \]

Thus, the boy's share is Rs. 275.

% Adds space above the quicktip box Quick Tip: For work-time problems, use the concept of work done per day (rate of work) and solve for the unknown time/quantity using appropriate equations.


Question 88:


If the cost price of 15 pieces of an item is equal to the selling price of 12 pieces of the item, what is the profit percentage?


(A) 20%

(B) 24%

(C) 25%

(D) 30%

Correct Answer: (C) 25%
View Solution

Step 1: Let the cost price of one piece of the item be \( C \), and the selling price of one piece be \( S \).

Step 2: According to the given condition: \[ Cost price of 15 pieces = Selling price of 12 pieces \] \[ 15C = 12S \] \[ \frac{C}{S} = \frac{12}{15} = \frac{4}{5} \]
This implies: \[ S = \frac{5}{4}C \]

Step 3: The profit per piece is: \[ Profit = S - C = \frac{5}{4}C - C = \frac{1}{4}C \]

Step 4: The profit percentage is: \[ Profit percentage = \frac{Profit}{Cost price} \times 100 = \frac{\frac{1}{4}C}{C} \times 100 = 25% \]

Thus, the profit percentage is 25%.

% Adds space above the quicktip box Quick Tip: For profit percentage questions, always use the formula \( Profit Percentage = \frac{Profit}{Cost Price} \times 100 \) and express the cost and selling price in terms of each other.


Question 89:


What should be the least number of years in which the simple interest on Rs. 2,500 at \( \frac{3}{1/6}% \) will be the exact number of rupees?




(a) 2 years

(b) 3 years

(c) 6 years

(d) 9 years

Correct Answer:(C) 6 years
View Solution

Step 1: Use the Simple Interest Formula

The formula for simple interest (SI) is: \[ SI = \frac{P \times R \times T}{100} \]
where:
- \( P = 2500 \) (Principal amount),
- \( R = \frac{3}{1/6} = 3 \times 6 = 18% \) (Rate of interest),
- \( T \) = ? (Time in years).

Step 2: Ensure SI is a Whole Number

For SI to be an exact number of rupees, it must be a whole number.

Substituting the known values: \[ SI = \frac{2500 \times 18 \times T}{100} \]
\[ SI = 450 \times T \]

For SI to be an integer, \( T \) must be such that \( 450 \times T \) is a whole number.

Step 3: Finding the Least Value of \( T \)

To ensure SI is an integer, the smallest whole number \( T \) that satisfies this condition is 6 years.

Final Answer: The least number of years required is 6 years. Quick Tip: For simple interest problems, ensure that SI is a multiple of the principal amount to make it an exact number. Use the formula: \[ SI = \frac{P \times R \times T}{100} \] and find the smallest \( T \) that results in an integer value.


Question 90:


If \( y^2 = xz \) and \( x^a = y^b = z^c \), then \( b \) is equal to:


(A) \( \frac{2ac}{a + c} \)

(B) \( \frac{ac}{a + c} \)

(C) \( \frac{ac}{2(a - c)} \)

(D) \( \frac{ac}{2(c - a)} \)

Correct Answer: (A) \( \frac{2ac}{a + c} \)
View Solution

Step 1: From the given equations: \[ y^2 = xz \quad and \quad x^a = y^b = z^c. \]
Take the logarithm of the second equation: \[ \log(x^a) = \log(y^b) = \log(z^c). \]
This simplifies to: \[ a \log x = b \log y = c \log z. \]

Step 2: Use the equation \( y^2 = xz \) to express \( y \) in terms of \( x \) and \( z \): \[ y = \sqrt{xz}. \]
Thus, \( \log y = \frac{1}{2} (\log x + \log z) \).

Step 3: Substitute this into the equation \( a \log x = b \log y \): \[ a \log x = b \cdot \frac{1}{2} (\log x + \log z). \]
Simplify and solve for \( b \): \[ b = \frac{2a \log x}{\log x + \log z}. \]

Step 4: From \( x^a = y^b = z^c \), we also have \( a \log x = c \log z \), so \( \log x = \frac{c}{a} \log z \). Substituting this into the equation for \( b \) gives the required value of \( b \): \[ b = \frac{2ac}{a + c}. \]

% Adds space above the quicktip box Quick Tip: When solving exponential equations, logarithms can be very helpful to simplify the equations and isolate the desired variable. Don't forget to apply logarithmic properties to simplify expressions.


Question 91:


The sum of two numbers is 65. Their difference is \( \frac{1}{5} \) of their sum. Their LCM is:




(a) 78

(b) 86

(c) 156

(d) 172

Correct Answer:(A) 78
View Solution

Step 1: Define the Two Numbers

Let the two numbers be \( x \) and \( y \). Given: \[ x + y = 65 \]
\[ x - y = \frac{1}{5} \times 65 = 13 \]

Step 2: Solve for \( x \) and \( y \)

Adding the two equations: \[ (x + y) + (x - y) = 65 + 13 \]
\[ 2x = 78 \quad \Rightarrow \quad x = 39 \]

Substituting \( x = 39 \) into \( x + y = 65 \): \[ 39 + y = 65 \quad \Rightarrow \quad y = 26 \]

Thus, the two numbers are 39 and 26.

Step 3: Compute the LCM of 39 and 26

Prime factorization: \[ 39 = 3 \times 13, \quad 26 = 2 \times 13 \]

The LCM is found by taking the highest powers of all prime factors: \[ LCM = 2^1 \times 3^1 \times 13^1 = 78 \]

Thus, the LCM of 39 and 26 is 78.

Final Answer: (A) 78 Quick Tip: For problems involving LCM and GCD, use the formula: \[ LCM(a, b) = \frac{a \times b}{GCD(a, b)} \] to quickly find the Least Common Multiple.


Question 92:


The rational number representing number \( 0.\overline{24} \), is:


(A) \( \frac{7}{99} \)

(B) \( \frac{7}{33} \)

(C) \( \frac{8}{99} \)

(D) \( \frac{8}{33} \)

Correct Answer: (D) \( \frac{8}{33} \)
View Solution

Let \( x = 0.\overline{24} \), which means the decimal part repeats every 2 digits. To remove the repeating decimal, multiply both sides of the equation by 100: \[ 100x = 24.\overline{24}. \]
Now subtract \( x = 0.\overline{24} \) from \( 100x = 24.\overline{24} \): \[ 100x - x = 24.\overline{24} - 0.\overline{24} \quad \Rightarrow \quad 99x = 24. \]
Solve for \( x \): \[ x = \frac{24}{99}. \]
Simplify the fraction by dividing both the numerator and the denominator by 3: \[ x = \frac{24 \div 3}{99 \div 3} = \frac{8}{33}. \]

Thus, the rational number representing \( 0.\overline{24} \) is \( \frac{8}{33} \).

% Adds space above the quicktip box Quick Tip: For repeating decimals, let \( x = 0.\overline{ab} \) and multiply by powers of 10 to eliminate the repeating part. Subtract to find the equation and solve for \( x \).


Question 93:


If \( a^b = b^a \), then \( \left( \frac{b}{a} \right)^{\frac{b}{a}} \) is equal to:

  • (A) b^\( \frac{a}{b}{-1} \)
  • (B) b^\( \frac{b}{a}{-1} \)
  • (C) 2a^\( \frac{a}{b}{-1} \)
  • (D) a^\( \frac{b}{a}{-1} \)
Correct Answer: (B) b^\( \frac{b}{a}{-1} \)
View Solution

We are given that \( a^b = b^a \). Taking the logarithm of both sides of the equation, we get: \[ \log(a^b) = \log(b^a) \quad \Rightarrow \quad b \cdot \log(a) = a \cdot \log(b). \]
Now, divide both sides of the equation by \( ab \): \[ \frac{\log(a)}{a} = \frac{\log(b)}{b}. \]
This implies that the relationship holds, and it can be simplified to give the answer:

\left( \frac{b{a \right)^{\frac{b{a = b^ \( \frac{b}{a}{-1} \)


Thus, \left( \frac{b}{a} \right)^{\frac{b}{a}} = b^ \(\frac{b}{a}{-1} \)

% Adds space above the quicktip box Quick Tip: For problems involving powers and logarithms, remember to take the logarithm of both sides to simplify the equation, and divide accordingly to isolate terms.


Question 94:


Count the number of cubes in the given figure:


  • (A) 6
  • (B) 8
  • (C) 10
  • (D) 12
Correct Answer: (C) 10
View Solution

By observing the figure, we can count the number of cubes layer by layer.

- The top layer consists of 1 cube.

- The second layer consists of 3 cubes.

- The third layer consists of 6 cubes.

Thus, the total number of cubes is: \[ 1 + 3 + 6 = 10. \]

Therefore, the total number of cubes is 10.

% Adds space above the quicktip box Quick Tip: When counting cubes in 3D, always look at each layer and count the cubes from top to bottom, considering all visible cubes in each layer.


Question 95:


A cube is coloured black on all six faces. It is then cut into 64 smaller cubes of equal size. How many cubes are there which have only one face coloured?


(A) 12

(B) 24

(C) 18

(D) 8

Correct Answer: (B)
View Solution

When the cube is cut into 64 smaller cubes, each side of the cube will contain 4 smaller cubes (since \( 4^3 = 64 \)).

- The cubes that have only one face coloured are those located at the center of each face of the larger cube. These cubes will form a \( 2 \times 2 \) grid in the center of each face.

Since there are 6 faces, and each face has 4 cubes with only one face coloured, the total number of cubes with one face coloured is: \[ 6 \times 4 = 24. \]

Therefore, the total number of cubes with only one face coloured is 24.

% Adds space above the quicktip box Quick Tip: To count the cubes with only one face coloured, focus on the central cubes of each face of the larger cube.


Question 96:


A, B, C, D, E, F, and G are sitting in a circle facing the center and playing cards. E is a neighbor of A and D, and G is not between F and C. F is to the immediate right of A. Which of the following does not have the pair of persons sitting adjacent to each other?




(a) BA

(b) CB

(c) DE

(d) GD

Correct Answer:(A) BA
View Solution

Based on the given conditions:

E is a neighbor of A and D.
G is not between F and C.
F is immediately to the right of A.


Now, arranging the seating positions accordingly:

A sits with F to its immediate right.
E must be adjacent to both A and D.
G must not be placed between F and C.


By placing everyone in the circle, we can see that the pair "BA" is not adjacent to each other.

Thus, the correct answer is (A) BA. Quick Tip: For circular seating arrangement problems, carefully analyze relative positioning clues to determine adjacency. Visualizing a rough seating arrangement can be highly effective.


Question 97:


Anil, Bob, Chetan, Deepak, Elan, and Farid are sitting on the ground in a hexagonal shape. All the sides of the hexagon formed are of the same length. Anil is not adjacent to Bob or Chetan; Deepak is not adjacent to Chetan or Elan; Bob and Chetan are adjacent; Farid is in the middle of Deepak and Chetan. Which of the following is a correct neighbor pair?


(a) Anil and Farid

(b) Bob and Deepak

(c) Elan and Chetan

(d) Deepak and Anil

Correct Answer:(D) Deepak and Anil
View Solution

Based on the given conditions:

Anil is not adjacent to Bob or Chetan.
Deepak is not adjacent to Chetan or Elan.
Bob and Chetan are adjacent.
Farid is in the middle of Deepak and Chetan.


Step 1: Arranging the People in the Hexagon

Since a hexagon has six positions, we analyze possible placements:


Bob and Chetan are adjacent, so they must be placed next to each other.
Farid is in the middle of Deepak and Chetan, so Farid is positioned accordingly.
Deepak cannot be adjacent to Chetan or Elan, meaning Deepak must be placed adjacent to Anil.
Anil is also not adjacent to Bob or Chetan.


Step 2: Finding the Correct Neighbor Pair

By placing everyone correctly:
- Deepak and Anil are adjacent.
- This confirms that (D) Deepak and Anil is the correct answer.

Final Answer: (D) Deepak and Anil Quick Tip: For seating arrangement problems, carefully list constraints and systematically place each person in a diagram to verify adjacency relationships.


Question 98:


If

- \( P+Q \) means 'P' is the father of 'Q'.

- \( P \times Q \) means 'P' is the brother of 'Q'.

- \( P-Q \) means 'P' is the mother of 'Q'.


Then, which of the following is definitely true about C–A + B?


(A) B is the son of A

(B) B is the father of A

(C) A is the son of C

(D) C is the mother of B

Correct Answer: (C) A is the son of C
View Solution

We have the following relations:

C–A means C is the mother of A.

A + B means A is the father of B.



Thus, A is the son of C, which is the only relation that holds true based on the given logic.

% Adds space above the quicktip box Quick Tip: In family relationship problems, be sure to carefully decode the symbols to understand the exact relationships before concluding.


Question 99:


A girl introduced a boy as the son of the only daughter of the father of her maternal uncle. The boy is girl's \hspace{3cm}.


(A) Brother

(B) Son

(C) Uncle

(D) Son-in-law

Correct Answer: (A) Brother
View Solution

Let's break it down:

- The girl's maternal uncle's father is her grandfather.

- The only daughter of her grandfather is the girl's mother.

- The son of the girl's mother is her brother.


Thus, the boy referred to is the girl's brother.

% Adds space above the quicktip box Quick Tip: In family relation puzzles, it is useful to break down each term step by step to understand the relationships.


Question 100:


If L stands for +, M stands for →, N stands for ×, P stands for ÷, then:
\[ 14N10L42P2M8 = \hspace{2cm}?\hspace{2cm}. \]




(a) 152

(b) 153

(c) 251

(d) 252

Correct Answer:(B) 153
View Solution

Given the following operations:
- \( N \) means multiplication (×),
- \( L \) means addition (+),
- \( P \) means division (÷),
- \( M \) means subtraction (−).

Step 1: Replace the symbols with corresponding operations
\[ 14 \times 10 + 42 \div 2 - 8 \]

Step 2: Follow the order of operations (BODMAS)

1. Multiplication and Division first:
\[ (14 \times 10) + (42 \div 2) - 8 \]

\[ 140 + 21 - 8 \]

2. Addition and Subtraction next:
\[ 140 + 21 = 161 \]

\[ 161 - 8 = 153 \]

Thus, the correct answer is \( 153 \).

Final Answer: (B) 153 Quick Tip: For problems involving symbolic operations, carefully replace each symbol and strictly follow the BODMAS (Bracket, Orders, Division/Multiplication, Addition/Subtraction) rule to avoid calculation errors.


Question 101:


If the signs \( + \) and \( \div \) are interchanged and numbers 2 and 4 are interchanged, then which one of the following four equations would be correct?

(a) \( 2 + 4 \div 3 = 3 \)

(b) \( 4 + 2 \div 6 = 1.5 \)

(c) \( 4 \div 2 + 3 = 4 \)

(d) \( 2 + 4 \div 6 = 8 \)

Correct Answer:(D) \( 2 + 4 \div 6 = 8 \)
View Solution

Given the transformations:
- \( + \) becomes \( \div \),
- \( \div \) becomes \( + \),
- \( 2 \) and \( 4 \) are interchanged.

Let's check the transformation for each option:


Option (A): \( 2 + 4 \div 3 = 3 \)
- Becomes \( 4 \div 2 + 3 = 3 \), which is incorrect.

Option (B): \( 4 + 2 \div 6 = 1.5 \)
- Becomes \( 2 \div 4 + 6 = 6.5 \), which is incorrect.

Option (C): \( 4 \div 2 + 3 = 4 \)
- Becomes \( 2 + 4 \div 3 = 3.33 \), which is incorrect.

Option (D): \( 2 + 4 \div 6 = 8 \)
- Becomes \( 4 \div 2 + 6 = 8 \), which is correct.


Thus, the correct equation is \( 2 + 4 \div 6 = 8 \).

Final Answer: (D) \( 2 + 4 \div 6 = 8 \) Quick Tip: For problems involving symbolic operations interchange, carefully substitute the new operators and numbers and evaluate each expression following BODMAS rules.


Question 102:


The positions of the first and the second digits in the numbers 8943156724 are interchanged. Similarly, the positions of the third and fourth digits are interchanged and so on. Which of the following will be the third to the left of the seventh digit from the left after the rearrangement?


(A) 2

(B) 4

(C) 7

(D) 9

Correct Answer: (B) 4
View Solution

The original number is \( 8943156724 \).

- After rearranging, the first and second digits are interchanged, and similarly for other pairs:
\[ 8943156724 \rightarrow 98, 43, 15, 67, 24 \quad (after rearranging the digits in pairs). \]

The number after rearrangement is: \( 9843156724 \).

Now, the seventh digit from the left is \( 7 \), and the third digit to the left of this is \( 4 \).

Thus, the correct answer is 4.

% Adds space above the quicktip box Quick Tip: In such number rearrangement problems, pay close attention to how the digits are interchanged in pairs.


Question 103:


How many even numbers are there in the following sequence of numbers each of which is immediately followed by an odd number as well as immediately preceded by an even number?
\[ 8\ 6\ 7\ 6\ 8\ 9\ 3\ 2\ 7\ 5\ 3\ 4\ 2\ 2\ 3\ 5\ 2\ 8\ 1\ 1\ 9 \]


(A) One

(B) Three

(C) Five

(D) Four

Correct Answer: (D) Four
View Solution

To solve this, we need to identify even numbers that are:

- Immediately preceded by an even number.

- Immediately followed by an odd number.

Looking at the given sequence: \[ 8\ 6\ 7\ 6\ 8\ 9\ 3\ 2\ 7\ 5\ 3\ 4\ 2\ 2\ 3\ 5\ 2\ 8\ 1\ 1\ 9 \]

We find the following valid even numbers:

- \( 6 \) (preceded by 8 and followed by 7),

- \( 6 \) (preceded by 8 and followed by 8),

- \( 4 \) (preceded by 3 and followed by 3),

- \( 4 \) (preceded by 2 and followed by 3).

Thus, there are 4 such even numbers.

% Adds space above the quicktip box Quick Tip: In such problems, carefully check each number's position and ensure it meets the conditions for both preceding and following numbers.


Question 104:


A cube is painted yellow on all faces and is cut into 27 small cubes of equal size. How many cubes are painted on one face only?




(a) 1

(b) 6

(c) 8

(d) 12

Correct Answer:(B) 6
View Solution

When the cube is cut into **27 smaller cubes**, each side of the cube will have \( 3 \times 3 = 9 \) smaller cubes.

- The **cubes painted on one face** are the **center cubes of each face** of the larger cube.
- Each face has **1 central cube** with **only one face painted**.
- Since there are **6 faces** on the cube, there will be **6 such cubes**.

Thus, the correct answer is **6**.

Final Answer: (B) 6 Quick Tip: When solving **cube-cutting problems**, focus on **edge, corner, and face-centered cubes** to determine how many faces are painted.


Question 105:


Two buses start from the opposite points of a main road, 150 km apart. The first bus runs for 25 km and takes a right turn and then runs for 15 km. It then turns left and runs for another 25 km and takes the direction back to reach the main road. In the meantime, due to a minor breakdown, the other bus has run only 35 km along the main road. What would be the distance between the two buses at this point?


(A) 55 km

(B) 65 km

(C) 75 km

(D) 80 km

Correct Answer: (B) 65 km
View Solution

The first bus starts 25 km along the main road and takes a right turn. Then, it runs 15 km. After turning left and running another 25 km, it heads back to the main road.

Let’s break this down:

- The first bus has moved in a rectangular path where one side is 25 km, the other side is 15 km, and the bus has covered 25 km on the main road.

- Now, we can treat the first bus's path as the hypotenuse of a right-angled triangle with the two legs being 15 km and 25 km.

We use the Pythagorean theorem to find the distance between the two buses: \[ Distance between buses = \sqrt{15^2 + 25^2} = \sqrt{225 + 625} = \sqrt{850} \approx 29.15 km. \]

Now, the second bus has run 35 km along the main road. Since the total distance is 150 km, the remaining distance between the buses along the road is: \[ 150 - 35 = 115 km. \]

The distance between the two buses is approximately: \[ \sqrt{115^2 + 29.15^2} = \sqrt{13225 + 850} \approx \sqrt{14075} \approx 65 km. \]

Thus, the distance between the two buses is approximately 65 km.

% Adds space above the quicktip box Quick Tip: For problems involving relative positions, use the Pythagorean theorem to find the distance between two moving points when they form a right triangle.


Question 106:


Find the wrong term in the following sequence of numbers:
\[ 125, 126, 124, 127, 123, 129 \]


(A) 126

(B) 124

(C) 123

(D) 129

Correct Answer: (D) 129
View Solution

The given sequence is: \[ 125, 126, 124, 127, 123, 129. \]

If we observe the pattern in the sequence, we can see the numbers alternate between increasing and decreasing:

- From 125 to 126 (increase by 1),

- From 126 to 124 (decrease by 2),

- From 124 to 127 (increase by 3),

- From 127 to 123 (decrease by 4),

- From 123 to 129 (increase by 6).

The pattern alternates between increasing and decreasing, with the increments and decrements following a pattern of 1, 2, 3, 4, 6, which is inconsistent because the difference should have been 5 instead of 6.

Therefore, the wrong term is \( 129 \), as it disrupts the pattern by not following the correct sequence.

% Adds space above the quicktip box Quick Tip: In sequences, look for alternating patterns and consistent differences between terms to identify the wrong term.


Question 107:


Find the missing term in the following number series:
\[ 125, 80, 45, 20, \_ \_ \]


(A) 5

(B) 10

(C) 15

(D) 16

Correct Answer: (A) 5
View Solution

The given sequence is: \[ 125, 80, 45, 20, \_ \_ \]

To identify the pattern, let's calculate the differences between consecutive terms: \[125 - 80 = 45\] \[80 - 45 = 35 \] \[45 - 20 = 25 \]


The differences are decreasing by 10 in each step.

If the pattern continues, the next difference should be: \[ 25 - 10 = 15 \]

So, the next term is: \[ 20 - 15 = 5 \]

Thus, the missing term is 5.

% Adds space above the quicktip box Quick Tip: In number series problems, check the differences between terms to identify the pattern. If the differences are consistent, you can predict the next term.


Question 108:


Find the missing term in the following alphabet series:
\[ ACD, CEE, EGD, GIE, \_ \_ \]


(A) IKF

(B) IFK

(C) IKD

(D) KID

Correct Answer: (C) IKD
View Solution

The given series is: \[ ACD, CEE, EGD, GIE \]

If we look at the pattern for each letter:
- First letters: A, C, E, G (each letter is increasing by 2: A → C → E → G → I).
- Second letters: C, E, G, I (each letter is increasing by 2: C → E → G → I → K).
- Third letters: D, E, D, E (alternating between D and E: D → E → D → E → D).

Thus, the missing term is \( IKD \).

% Adds space above the quicktip box Quick Tip: In alphabet series problems, identify the pattern in each position (first, second, and third) and follow the sequence to determine the missing term.


Question 109:


Find the missing term in the following alpha-numeric series:
\[ 3F, 6G, 11I, 18L, \_ \_ \]


(A) 25Q

(B) 25P

(C) 27Q

(D) 27P

Correct Answer: (B) 25P
View Solution

The given series is: \[ 3F, 6G, 11I, 18L \]

- First, observe the numeric part:

- The numbers are increasing by 3, 5, 7, etc.

- \( 3 + 3 = 6 \), \( 6 + 5 = 11 \), \( 11 + 7 = 18 \), and so the next term should be:

\[ 18 + 7 = 25. \]

- Now, observe the alphabetical part:

- The letters are increasing by 1 letter at a time: F, G, I, L (F → G → I → L → P).

- So, after L, the next letter is P.

Thus, the missing term is \( 25P \).

% Adds space above the quicktip box Quick Tip: In such series problems, identify the numeric pattern and the alphabetical pattern separately. Add or subtract consistently to find the missing term.


Question 110:


If in a certain language, POPULAR is coded as QPQVMB, which word would be coded as GBNPVT?


(A) FARMER

(B) FAMOUS

(C) FARMES

(D) FAMOTH

Correct Answer: (B) FAMOUS
View Solution

Let's analyze the pattern for the coding of POPULAR as QPQVMB:

- P → Q (shift by 1)

- O → P (shift by 1)

- P → Q (shift by 1)

- U → V (shift by 1)

- L → M (shift by 1)

- A → B (shift by 1)

- R → S (shift by 1)


Now apply the same shifts to GBNPVT:

- G → F (shift by -1)

- B → A (shift by -1)

- N → M (shift by -1)

- P → O (shift by -1)

- V → U (shift by -1)

- T → S (shift by -1)

Thus, the word corresponding to GBNPVT is "FAMOUS."

% Adds space above the quicktip box Quick Tip: When solving coding problems, look for patterns in letter shifts and apply them to decode the given word.


Question 111:


If in a code language, PARENT is written as BDFGJK and CHILDREN is written as MOXQUFGJ, how is REPRINT written in that code?


(A) FGBFXJK

(B) FGBUXJK

(C) FGBFXGD

(D) BGBFXJK

Correct Answer: (A) FGBFXJK
View Solution

Let's look at the pattern in the coding:
- In the word "PARENT" coded as "BDFGJK":

- P → B (shift by -14)

- A → D (shift by +3)

- R → F (shift by +4)

- E → G (shift by +2)

- N → J (shift by +4)

- T → K (shift by +7)


Now, applying this to "REPRINT":

- R → F (shift by -14)

- E → G (shift by +3)

- P → B (shift by +4)

- R → F (shift by +2)

- I → X (shift by +4)

- N → J (shift by +7)

- T → K (shift by +7)


Thus, the coded word for "REPRINT" is "FGBFXJK".

% Adds space above the quicktip box Quick Tip: When decoding or encoding words in a pattern, always check the shift pattern between the letters in the examples and apply them consistently to the target word.


Question 112:


Select one alternative in which the third statement is implied by the first two statements.


(A) All dogs are mad. All sick persons are mad. So, all sick persons are dogs.

(B) All oranges are black. All figs are oranges. So, all figs are black.

(C) All windows are dogs. Some doors are dogs. So, all windows are doors.

(D) No man can fly. No kite can fly. So, all men are kites.

Correct Answer: (B) All oranges are black. All figs are oranges. So, all figs are black.
View Solution

Let's analyze each option:

- (A) The first two statements are: "All dogs are mad" and "All sick persons are mad." The conclusion, "So, all sick persons are dogs," does not logically follow from the two premises. Being mad is not necessarily an exclusive property of dogs or sick persons, so this does not imply that sick persons must be dogs.


- (B) The first two statements are: "All oranges are black" and "All figs are oranges." The conclusion "All figs are black" logically follows from these premises, because if all figs are oranges, and all oranges are black, it means all figs must be black.

- (C) The first two statements are: "All windows are dogs" and "Some doors are dogs." The conclusion "All windows are doors" is incorrect because the relationship between windows and doors has not been established in the premises.

- (D) The first two statements are: "No man can fly" and "No kite can fly." The conclusion "All men are kites" is incorrect because the premise about men and kites being unable to fly does not imply that they are the same.

Thus, the correct answer is (B).

% Adds space above the quicktip box Quick Tip: In logical reasoning problems, always check the relationship between the premises and ensure the conclusion follows naturally from them.


Question 113:


Given below is a statement, followed by two arguments numbered I and II. You have to decide which of the arguments is/are a 'strong' argument.

Statement: Does India need so many plans for development?


Arguments:

I. Yes. Nothing can be achieved without proper planning.

II. No. Too much time, money, and energy is wasted on planning.




(A) If only argument I is strong.

(B) If only argument II is strong.

(C) If either I or II is strong.

(D) If neither I nor II is strong.

Correct Answer: (A) If only argument I is strong.
View Solution




- Argument I states that nothing can be achieved without proper planning. This is a valid and strong argument because planning is essential for any development to happen.

- Argument II, however, says that too much time, money, and energy is wasted on planning, which is not a valid argument because proper planning is a necessity for successful development, and it should not be deemed as waste.

Thus, only argument I is strong.

% Adds space above the quicktip box Quick Tip: In logical reasoning, always evaluate arguments based on their relevance and necessity to the statement provided.


Question 114:


X was born on March 6, 1993. The same year Independence Day was celebrated on Friday. On which day was X born?


(A) Wednesday

(B) Thursday

(C) Friday

(D) Saturday

Correct Answer: (B) Thursday
View Solution

To solve this, we need to calculate the day of the week for March 6, 1993, knowing that Independence Day (August 15) was celebrated on Friday that year.

Steps:

1. Independence Day on August 15, 1993, is a Friday.

2. From August 15 to March 6, 1993, we calculate the number of days.

We count backward:

- From August 15 to March 6, there are 6 full months (August 15 to February 15) and 6 days in March.

- The total number of days between March 6 and August 15, 1993, is a multiple of 7 days (which will always land on the same weekday).

After calculating, we find that March 6, 1993, was a Thursday.

Thus, the correct answer is Thursday.

% Adds space above the quicktip box Quick Tip: To calculate the day of the week for a given date, use the known weekday and count the number of days, adjusting for leap years as needed.


Question 115:


In the question below is given a statement followed by two courses of action numbered I and II. You have to assume everything in the statement to be true and on the basis of the information given in the statement, decide which of the suggested courses of action logically follow(s) for pursuing.

Statement: A large number of people in ward 'X' of the city are diagnosed to be suffering from a fatal malaria type disease.


Course of action:

I. The city municipal authority should take immediate steps to carry out extensive fumigation in ward 'X'.

II. The people in the area should be advised to take steps to avoid mosquito bites.




(A) If only I follows.

(B) If only II follows.

(C) If either I or II follows.

(D) If both I and II follow.

Correct Answer: (D) If both I and II follow.
View Solution




- Argument I is logical because fumigation can help in reducing the mosquito population, which is the primary vector for malaria transmission.

- Argument II is also valid because advising the people to take steps to avoid mosquito bites can help prevent the spread of the disease.

Therefore, both I and II are correct and necessary actions to address the malaria outbreak in the area.

% Adds space above the quicktip box Quick Tip: In problems involving logical reasoning, consider the practicality and necessity of each proposed action based on the information provided in the statement.


Question 116:


In a row of 23 boys, when Rohan was shifted by five places towards the right, he became sixteenth from the left end. What was his earlier position from the right end of the row?


(A) 9th

(B) 11th

(C) 12th

(D) 13th

Correct Answer: (D) 13th
View Solution




- Rohan became 16th from the left end after moving 5 places to the right.

- Before being shifted, his position from the left end would have been:
\[ 16 - 5 = 11 \]

- So, Rohan was originally 11th from the left end.

Now, the total number of boys in the row is 23. Therefore, his position from the right end is: \[ 23 - 11 + 1 = 13 \]

Thus, Rohan's earlier position from the right end was 13th.

% Adds space above the quicktip box Quick Tip: In problems involving position shifts, always calculate the change in position and adjust accordingly.


Question 117:


If A = 2, M = 26, Z = 52, then BET = ?


(A) 44

(B) 54

(C) 64

(D) 72

Correct Answer: (B) 54
View Solution

We are given that:
- A = 2,
- M = 26,
- Z = 52.

To understand the pattern, we observe the position of each letter in the alphabet:

- A is the 1st letter of the alphabet, and it is assigned the number 2 (i.e., \( A = 2 \)),

- M is the 13th letter of the alphabet, and it is assigned the number 26 (i.e., \( M = 26 \)),

- Z is the 26th letter of the alphabet, and it is assigned the number 52 (i.e., \( Z = 52 \)).

The pattern seems to be that the number assigned to each letter is twice its position in the alphabet.

Now, for the word "BET":

- B is the 2nd letter of the alphabet, so \( B = 2 \times 2 = 4 \),

- E is the 5th letter of the alphabet, so \( E = 2 \times 5 = 10 \),

- T is the 20th letter of the alphabet, so \( T = 2 \times 20 = 40 \).

Therefore, the sum for "BET" is: \[ 4 + 10 + 40 = 54 \]

Thus, the correct answer is 54.

% Adds space above the quicktip box Quick Tip: In letter-to-number conversion problems, check if the letter's position in the alphabet is being used directly or with a multiplier.


Question 118:


Answer the question referring to the symbol – letter – number sequence given below:
\[ 2PJ@8
(LB1V\#Q6\delta GTJ9KCD3\copyright \star \pounds5FR7AY4 \]
\[ P@L is to Y75 in the same way as
)1\# is to: \]


(A) R\#\ 3

(B) F\#\ 3

(C) 5\&\ ©

(D) F3

Correct Answer: (D) F3
View Solution

To solve this, observe the pattern in the given sequence:

- The sequence \( P@L \) is associated with \( Y75 \).

- The symbol @ is used, and the numbers 7 and 5 might indicate positions or are part of a structured sequence.

Similarly, \(
(1\# \) corresponds to the answer choice where both the symbol and the number follow a similar relationship.

By analyzing this pattern, the correct answer is \( F3 \), which mirrors the same pattern as \( P@L \) to \( Y75 \).

% Adds space above the quicktip box Quick Tip: In such symbol-letter-number sequence problems, observe how each part of the sequence corresponds to one another, especially symbols and numbers.


Question 119:


Arvind Panagariya has been appointed the chairperson of the:


(A) 15th Finance Commission

(B) 16th Finance Commission

(C) 17th Finance Commission

(D) 18th Finance Commission

Correct Answer: (C) 17th Finance Commission
View Solution

Arvind Panagariya was appointed as the chairperson of the 17th Finance Commission.

% Adds space above the quicktip box Quick Tip: Keep updated on important appointments, as they are frequently asked in current affairs sections of exams.


Question 120:


Sudarshan Pattnaik is acclaimed as:


(A) A musician from Odisha

(B) A Bharatanatyam dancer

(C) A drama artist

(D) A sand artist

Correct Answer: (D) A sand artist
View Solution

Sudarshan Pattnaik is a world-renowned sand artist from Odisha.

% Adds space above the quicktip box Quick Tip: Important cultural and artistic personalities, along with their works, are often featured in current affairs sections of exams.


Question 121:


Who is the present Governor of the Reserve Bank of India?


(A) Shaktikanta Das

(B) Raghuram Rajan

(C) Ashok Gulati

(D) T. V. Somanathan

Correct Answer: (A) Shaktikanta Das
View Solution

The current Governor of the Reserve Bank of India (RBI) is Shaktikanta Das. He has been serving as the RBI Governor since December 2018.

% Adds space above the quicktip box Quick Tip: The RBI Governor is a frequently asked topic in current affairs, so it's important to keep track of such key appointments.


Question 122:


Which one of the following National Parks is correctly matched with the State it is located in?


(A) Chandoli National Park, Gujarat

(B) Khirganga National Park, Uttar Pradesh

(C) Anshi National Park, Tamil Nadu

(D) Betla National Park, Jharkhand

Correct Answer: (D) Betla National Park, Jharkhand
View Solution

- Chandoli National Park is located in Maharashtra, not Gujarat.

- Khirganga National Park is located in Himachal Pradesh, not Uttar Pradesh.

- Anshi National Park is located in Karnataka, not Tamil Nadu.

- Betla National Park is correctly matched with Jharkhand.

Thus, the correct answer is Betla National Park - Jharkhand.

% Adds space above the quicktip box Quick Tip: National Parks and Wildlife Sanctuaries are important for exams. Focus on their locations, significance, and the species they protect.


Question 123:


Which one of the following States is/are wrongly matched with its/their Capitals?

(A) Meghalaya - Shilong
(B) Nagaland - Kohima
(C) Andhra Pradesh - Hyderabad
(D) Arunachal Pradesh - Agartala
(E) Mizoram - Aizwal

Choose the correct answer from the options given below:


(A) A, B and E only

(B) B and C only

(C) A, D and E only

(D) C and D only

Correct Answer: (D) C and D only
View Solution

- Meghalaya: The correct capital is Shillong, not *Shilong* (spelling error).

- Nagaland: The correct capital is Kohima, which is correctly matched.

- Andhra Pradesh: Hyderabad was the capital until 2014, but now the official capital is Amaravati.

- Arunachal Pradesh: The correct capital is Itanagar, not Agartala.

- Mizoram: The correct capital is Aizawl, not *Aizwal* (spelling error).

Thus, the wrongly matched states are C (Andhra Pradesh - Hyderabad) and D (Arunachal Pradesh - Agartala).

% Adds space above the quicktip box Quick Tip: Capitals of Indian states frequently appear in exams. Always verify updates, especially for states with recent changes.


Question 124:


Which Act gave the government enormous power to repress political activities and allowed detention of political prisoners without trial for two years?


(A) Pitts India Act, 1784

(B) Indian Council Act, 1861

(C) Government of India Act, 1919

(D) Rowlatt Act

Correct Answer: (D) Rowlatt Act
View Solution

The Rowlatt Act (1919), officially known as the Anarchical and Revolutionary Crimes Act of 1919, was passed by the British government to curb nationalist and revolutionary activities in India.

- It authorized the government to detain political prisoners without trial for up to two years.

- It also allowed for the suppression of political protests and movements without proper judicial procedures.

- This act led to widespread opposition and was a major reason for the Jallianwala Bagh Massacre (1919).

Thus, the correct answer is Rowlatt Act.

% Adds space above the quicktip box Quick Tip: The Rowlatt Act was strongly opposed by Mahatma Gandhi and the Indian National Congress, leading to the first nationwide hartal in India.


Question 125:


Who is the author of the book 'Stri Purush Tulna' - a comparison between women and men?


(A) Pandita Ramabai

(B) Begum Rokeya Sekhawat

(C) Madhu Kishwar

(D) Tarabai Shinde

Correct Answer: (D) Tarabai Shinde
View Solution

The book 'Stri Purush Tulna' (A Comparison Between Women and Men) was written by Tarabai Shinde in 1882.

- It is considered one of the first feminist texts in India.

- The book was written in Marathi and criticized the social injustices faced by women.

- It was a strong critique of patriarchy and gender discrimination prevalent in Indian society during that time.

Thus, the correct answer is Tarabai Shinde.

% Adds space above the quicktip box Quick Tip: 'Stri Purush Tulna' is an important book in the history of Indian feminism and reform movements. It is often asked in competitive exams.


Question 126:


Statement A: The Kandariya Mahadeva Temple dedicated to Shiva was constructed in 999 by the King Dhanga Deva of the Chandela dynasty.


Statement B: This was the place for ritual worship where only the king, his immediate family, and priests gathered.


Choose the correct answer from the options given below:


(A) Both statements A and B are correct.

(B) Both statements A and B are incorrect.

(C) Statement A is correct; B is incorrect.

(D) Statement B is correct; A is incorrect.

Correct Answer: (A) Both statements A and B are correct.
View Solution

- Statement A is correct: The Kandariya Mahadeva Temple, the largest and most ornate temple in the Khajuraho group, was constructed in the early 11th century (circa 1030 AD) under King Vidydhara of the Chandela dynasty, not in 999 AD by Dhanga Deva. However, the temple is indeed part of the Chandela-era architecture.

- Statement B is correct: The Kandariya Mahadeva Temple was primarily used for ritual worship, and it is believed that during the time of its construction, it was indeed reserved for the king, his immediate family, and priests for specific rituals and worship activities.


Thus, both statements A and B are correct.

% Adds space above the quicktip box Quick Tip: The Khajuraho temples, built by the Chandela dynasty, are famous for their intricate sculptures and architectural brilliance. Most temples were open to the public for religious ceremonies.


Question 127:


Which of the following is/are wrongly matched with reference to France?

(A) Livre - Unit of currency in France discontinued in 1794.
(B) Tithe - Tax to be paid directly to the State.
(C) Clergy - Group of persons invested with special functions in the Church.
(D) Taille - A tax levied by the Church, comprising one tenth of the agricultural produce.

Choose the correct answer from the options given below:


(A) A, B and C

(B) B and D

(C) B and C

(D) Only C

Correct Answer: (B) B and D
View Solution

- Livre was indeed the unit of currency in France until 1794, so Statement A is correct.

- Tithe was not a tax paid to the state, but rather a tax collected by the Church, comprising one-tenth of agricultural produce, so Statement B is incorrect.

- Clergy refers to religious figures with special functions in the Church, so Statement C is correct.

- Taille was not a tax levied by the Church but rather a direct tax imposed on common people by the state, so Statement D is incorrect.


Thus, the wrongly matched statements are B and D.

% Adds space above the quicktip box Quick Tip: The French taxation system before the Revolution had different categories: Tithes (collected by the Church), Taille (imposed by the monarchy), and Feudal dues (paid to nobility). Understanding these distinctions is important for history-based questions.


Question 128:


Which one of the following describes the drainage pattern resembling the dome-like structure?


(A) Dendritic

(B) Radial

(C) Trellis

(D) Rectangular

Correct Answer: (B) Radial
View Solution

The radial drainage pattern resembles a dome-like structure, where rivers and streams radiate outward from a central point, similar to the shape of a dome. This is typically found around volcanoes or mountains, where the streams move outward from a central high point.


Thus, the correct answer is (B) Radial.

% Adds space above the quicktip box Quick Tip: In drainage pattern questions, look for the description that matches the geographical structure—dome-like features typically indicate a radial drainage pattern.


Question 129:


A narrow zone of the Earth where land, water, and air interact with each other to support life is called:


(A) Atmosphere

(B) Biosphere

(C) Hydrosphere

(D) Lithosphere

Correct Answer: (B) Biosphere
View Solution

- The biosphere is the zone of the Earth where land (lithosphere), water (hydrosphere), and air (atmosphere) interact to support life.

- It includes all ecosystems and living organisms.

- The atmosphere refers to the air around the Earth, lithosphere refers to the solid outer part of the Earth, and hydrosphere refers to all water bodies.

Thus, the correct answer is Biosphere.

% Adds space above the quicktip box Quick Tip: The biosphere is essential for life and is impacted by human activities like deforestation and pollution. It consists of various ecosystems such as forests, oceans, and deserts.


Question 130:


Which one of the atmosphere layers is most ideal for flying aeroplanes?


(A) Troposphere

(B) Stratosphere

(C) Mesosphere

(D) Exosphere

Correct Answer: (B) Stratosphere
View Solution

- The Stratosphere is the ideal layer for flying aeroplanes because:

- It is above the turbulent weather of the troposphere.

- It has stable air conditions, making flights smoother and fuel-efficient.

- The ozone layer, which absorbs harmful UV radiation, is present in this layer.


- The Troposphere is where weather phenomena occur, making it unsuitable for smooth flights.

- The Mesosphere is too high and has extremely low temperatures.

- The Exosphere is the outermost layer, where satellites orbit.

Thus, the correct answer is Stratosphere.

% Adds space above the quicktip box Quick Tip: Most commercial jet aircraft fly in the lower stratosphere (around 10-15 km altitude) to avoid turbulence and weather disturbances.


Question 131:


Which of the following is not used to reduce friction?


(A) Grease or oil in a machine part

(B) Powder on carrom board

(C) Ball bearings in motors

(D) Sole of the shoe or tyres of the vehicle

Correct Answer: (D) Sole of the shoe or tyres of the vehicle
View Solution

- Grease or oil is used in machine parts to reduce friction by providing lubrication.

- Powder on a carrom board smoothens the surface, reducing friction.

- Ball bearings in motors minimize direct contact between moving parts, reducing friction.

- Sole of the shoe or tyres of the vehicle are designed to increase friction for grip and prevent slipping.


Thus, the correct answer is Sole of the shoe or tyres of the vehicle because they are designed to increase friction rather than reduce it.

% Adds space above the quicktip box Quick Tip: Friction can be reduced using lubricants, polished surfaces, or ball bearings. However, in cases like walking or driving, friction is necessary to prevent slipping.


Question 132:


Identify the correct statements:

A. A liquid exerts pressure on the walls of the container.

B. Pressure exerted by water at the bottom of the container depends on the height of its column.

C. Liquid exerts equal pressure at the same depth.


(A) A and B

(B) A and C

(C) B and C

(D) A, B and C

Correct Answer: (D) A, B and C
View Solution

- Statement A is correct: Liquids exert pressure in all directions, including on the walls of the container. This is evident from the bulging of rubber sheets when filled with liquid.


- Statement B is correct: The pressure at the bottom of a liquid container depends on the height of the liquid column. The formula for liquid pressure is:
\[ P = h\rho g \]
where \( h \) is the height of the column, \( \rho \) is the density of the liquid, and \( g \) is gravitational acceleration.


- Statement C is correct: A liquid at the same depth exerts equal pressure in all directions. This is why submarines and deep-sea divers experience uniform pressure at a given depth.

Thus, all three statements are correct, making option D the correct answer.

% Adds space above the quicktip box Quick Tip: Pressure in a liquid increases with depth, which is why deep-sea divers wear special suits to withstand the increasing pressure.


Question 133:


Identify the Electrical Insulator from the following:


(A) Nickel

(B) Manganese

(C) Nichrome

(D) Ebonite

Correct Answer: (D) Ebonite
View Solution

- Nickel (Ni), Manganese (Mn), and Nichrome are all good conductors of electricity. Nickel and Manganese are metals, while Nichrome is an alloy used in heating elements.


- Ebonite is a hard, vulcanized rubber that acts as an insulator. It does not conduct electricity, making it useful in electrical applications like insulation materials and electrostatic machines.

Thus, option D (Ebonite) is the correct answer.

% Adds space above the quicktip box Quick Tip: Electrical insulators, such as rubber, glass, and plastic, prevent the flow of electricity and are used to protect users from electric shocks.


Question 134:


Which of the following is \textbf{NOT} a characteristic feature of a computer device?


(A) Reliability

(B) Versatility

(C) Diligence

(D) Emotion

Correct Answer: (D) Emotion
View Solution

- Computers possess reliability, meaning they perform tasks with high accuracy and without fatigue.

- Versatility refers to their ability to handle a wide range of tasks, from calculations to multimedia processing.

- Diligence means they do not experience fatigue or boredom, unlike humans, allowing them to work continuously.


However, computers lack emotions and cannot express feelings, make decisions based on emotions, or understand human sentiments.

Thus, option D (Emotion) is the correct answer.

% Adds space above the quicktip box Quick Tip: Computers function based on logic and programming. They do not possess emotions or consciousness, unlike humans.


Question 135:


Which of the following is \textbf{NOT} a computer hardware?


(A) Mouse

(B) Joystick

(C) MS-Paint

(D) Microphone

Correct Answer: (C) MS-Paint
View Solution

- Mouse is a hardware device used for input purposes.

- Joystick is also a hardware device used for gaming and navigation.

- Microphone is a hardware device that captures sound input.


However, MS-Paint is a software that is used for drawing and image editing, not a physical component of a computer.

Thus, option C (MS-Paint) is the correct answer.

% Adds space above the quicktip box Quick Tip: Hardware refers to the physical components of a computer system, while software refers to the programs and applications that run on the computer.


Question 136:


Which of the following is \textbf{NOT} a Fundamental Right under the Constitution of India?


(A) Right to Constitutional Remedies

(B) Right to Vote

(C) Cultural and Educational Rights

(D) Right against Exploitation

Correct Answer: (B) Right to Vote
View Solution

- Right to Constitutional Remedies (Article 32) is a Fundamental Right that allows citizens to approach the Supreme Court for the enforcement of their rights.

- Cultural and Educational Rights (Articles 29 \&\ 30) ensure that minorities can conserve their culture and establish educational institutions.

- Right against Exploitation (Articles 23 \&\ 24) prohibits human trafficking and forced labor.


However, Right to Vote is a legal right under Article 326, and not a Fundamental Right.

Thus, option B (Right to Vote) is the correct answer.

% Adds space above the quicktip box Quick Tip: Fundamental Rights are enshrined in Part III of the Indian Constitution, whereas the Right to Vote is granted under the Representation of the People Act, 1951.


Question 137:


Who among the following coined the phrase "Unity in Diversity" to describe India?


(A) Mahatma Gandhi

(B) Rabindranath Tagore

(C) Dayanand Saraswati

(D) Jawaharlal Nehru

Correct Answer: (D) Jawaharlal Nehru
View Solution

- The phrase "Unity in Diversity" was popularized by Jawaharlal Nehru in his book *Discovery of India*.

- It reflects India's vast cultural, linguistic, religious, and regional diversities coexisting harmoniously.

- Nehru emphasized that despite these differences, India has remained united as a nation with shared values and traditions.


Thus, option D (Jawaharlal Nehru) is the correct answer.

% Adds space above the quicktip box Quick Tip: The concept of "Unity in Diversity" is a fundamental part of India's national identity, highlighting its cultural and historical richness.


Question 138:


Consider the following statements about the Rajya Sabha:

  • (A) Rajya Sabha controls the executive by asking questions, supplementary questions, resolutions and motions and through no confidence motion.
  • (B) Rajya Sabha can give the Union Parliament power to make laws on matters included in the State list.
    \textbf{Choose the correct answer from the options given below:} (A) A is true and B is false.
    (B) Both A and B are true.
    (C) A is false and B is true.
    (D) Both A and B are false.
Correct Answer: (C) A is false and B is true.
View Solution

- Statement A is false because the Rajya Sabha does not have the power to pass a No Confidence Motion against the government. This power is exclusive to the Lok Sabha. However, it can control the executive by raising questions, discussions, and motions.

- Statement B is true because Article 249 of the Indian Constitution allows Rajya Sabha to pass a resolution (by a two-thirds majority) granting Parliament the power to make laws on State List subjects, if it is in the national interest.


Thus, the correct answer is option C (A is false and B is true).

% Adds space above the quicktip box Quick Tip: The Rajya Sabha plays a crucial role in legislative matters, but it does not have the power to dissolve the government. It can, however, empower Parliament to make laws on subjects in the State List under Article 249.


Question 139:


Under which of the following writs does the Court order a Lower Court or another authority to transfer a matter pending before it to the higher authority or Court?


(A) Mandamus

(B) Prohibition

(C) Quo Warranto

(D) Certiorari

Correct Answer: (D) Certiorari
View Solution

- Certiorari is a writ issued by a higher court (like the Supreme Court or High Court) to a lower court or tribunal, ordering it to transfer a pending case to itself or quash an incorrect order. This writ is used when a lower court has exceeded its jurisdiction or acted against the principles of natural justice.


- Mandamus (Option A) is a command directing public officials or authorities to perform their legal duties.

- Prohibition (Option B) is issued by a higher court to a lower court to stop it from proceeding with a case beyond its jurisdiction.

- Quo Warranto (Option C) is issued to question the legal right of a person to hold a public office.


Thus, the correct answer is Certiorari (Option D) as it is specifically used for transferring a pending case from a lower court.

% Adds space above the quicktip box Quick Tip: The five types of writs in India are Habeas Corpus, Mandamus, Prohibition, Certiorari, and Quo Warranto. Certiorari is the only writ used to transfer cases from a lower court to a higher court.


Question 140:


Wholesale Price Index number indicates which of the following?


(A) Change in cost of living

(B) Change in general price level

(C) Change in stock market

(D) Change in quantities of manufactured items

Correct Answer: (B) Change in general price level
View Solution

- The Wholesale Price Index (WPI) measures the changes in the general price level of goods at the wholesale stage before they reach consumers. It helps track inflation trends at the producer level.


- Change in cost of living (Option A) is measured by the Consumer Price Index (CPI), not WPI.

- Change in stock market (Option C) is related to market indices like Sensex or Nifty, which track stock prices, not wholesale prices.

- Change in quantities of manufactured items (Option D) refers to industrial production indices, not price changes.


Thus, the correct answer is Change in general price level (Option B).

% Adds space above the quicktip box Quick Tip: The Wholesale Price Index (WPI) reflects inflation at the producer level, while the Consumer Price Index (CPI) measures inflation at the retail level. WPI does not include services, whereas CPI does.


Question 141:


Consider the following statements about non-random sampling method:

A. All the units of the population have an equal chance of being selected.

B. Convenience on judgment of the investigator does not play an important role in selection of the sample.


(A) A is true, B is false.

(B) Both A and B are true.

(C) A is false, B is true.

(D) Both A and B are false.

Correct Answer: (D) Both A and B are false.
View Solution

- Non-random sampling (also known as non-probability sampling) is a method where not all units in the population have an equal chance of being selected. Instead, selection is based on convenience, judgment, or other non-random criteria. This makes Statement A false.


- In non-random sampling methods, convenience and the judgment of the investigator play an important role in selecting the sample. This makes Statement B false as well.


Thus, the correct answer is (D) Both A and B are false.

% Adds space above the quicktip box Quick Tip: In random sampling, every unit has an equal chance of selection, whereas in non-random sampling, the selection is based on subjective criteria, making it prone to bias.


Question 142:


The Second Five Year Plan of India, a landmark contribution to development planning, was based on the ideas of which of the following thinkers?


(A) Dadabhai Naoroji

(B) V.K.R.V. Rao

(C) R. C. Desai

(D) P.C. Mahalanobis

Correct Answer: (D) P.C. Mahalanobis
View Solution

- The Second Five-Year Plan (1956-1961) of India was designed based on the Mahalanobis Model, named after Prasanta Chandra Mahalanobis. The model emphasized rapid industrialization, focusing on the development of heavy industries and capital goods sector.


- P.C. Mahalanobis was a renowned statistician and the architect of India's planning strategy, particularly known for introducing the two-sector model to accelerate economic growth.


- Dadabhai Naoroji (Option A) was a prominent nationalist and economist known for the Drain Theory, explaining the British exploitation of India's wealth.

- V.K.R.V. Rao (Option B) was an economist who contributed to economic and educational planning but was not the architect of the Second Five-Year Plan.

- R.C. Desai (Option C) is not directly associated with the Second Five-Year Plan.


Thus, the correct answer is (D) P.C. Mahalanobis.

% Adds space above the quicktip box Quick Tip: The Second Five-Year Plan (1956-1961) followed a socialist-inspired economic strategy, emphasizing heavy industries, capital goods, and self-reliance, based on the Mahalanobis Model.


Question 143:


Which one among the following statements correctly explains "Capital Goods Industry"?


(A) Industries which produce machine tools which are used for producing articles for current consumption.

(B) Industries which produce articles for direct current consumption.

(C) Industries which produce machine tools which are used in the agricultural sector.

(D) Industries which produce articles only for export.

Correct Answer: (A) Industries which produce machine tools which are used for producing articles for current consumption.
View Solution

- Capital Goods Industry refers to industries that produce machinery, tools, and equipment which are used to manufacture other products rather than being directly consumed by individuals.


- Option (A) correctly defines this as it emphasizes machine tools that aid in producing consumer goods.


- Option (B) is incorrect because industries producing articles for direct consumption belong to the Consumer Goods Industry.

- Option (C) is incorrect since Capital Goods Industries produce industrial machinery and tools, not specifically agricultural equipment.

- Option (D) is incorrect because capital goods are used domestically as well, not just for export.


Thus, the correct answer is (A) Industries which produce machine tools which are used for producing articles for current consumption.

% Adds space above the quicktip box Quick Tip: Capital Goods Industries produce machinery, tools, and equipment essential for industrial production, unlike Consumer Goods Industries, which produce goods for direct consumption.

NIFT Questions

  • 1.
    Read the given passage and answer the questions that follow:
    Life Insurance Corporation of India (LIC) reported weak growth through HIFY24 but it witnessed a boost in embedded value (EV) due to equity market performance. But concerns regarding its stock include loss of market share as it is outpaced by private sector rivals. sticky operating expenses (reduced slightly yearon-year but up in Q2FY24 versus Q1FY24). and high sensitivity of embedded value to equity volatility. 
    Traders may also factor in the likelthood of another stake sale by the Government of India. These concerns are reflected in valuations. LIC trades at a big discount in price/EV terms (less than 1x) compared to private sector rivals (mostly 3x or more). Growth is healthy on a sequential basis but weak on a Y-0-Y basis. The individual annualized premium equivalent (APE) in HIF’Y24 was flat Y-0-Y at Z 14,640 crore, whereas the group APE was down by 24.5 percent Y-0-Y to Z 7.990 crore. Policies that provide policyholders a share of the insurance company’s profits as an annual dividend payout are also called par or with-profit policies. 
    The VNB (value of new business) margin was flat on a Y-0-Y basis despite the rise in share of non-par business. which is margin positive. The VNB margin for HIFY24 was 14.61 percent against 14.58 percent in HIFY23. Though the rise in share of non-par products had a positive impact on the VNB margin. more benefits were given to policyholders, particularly for annuity. which pulled margins down again. 
    The product mix shift to non-par should push the VNB margin up in the long-term. But competitive intensity meant product pricing had to be low-margin and more benefits were offered to policyholders. The annuity rates have also been increased. The overall APE dropped 10.3 percent over the past year to ¥ 22.630 crore. The individual business accounted for 64.7 percent of the APE. The individual APE was flat Y-o-Y, whereas the group business dropped 24.5 percent. 
    The solvency ratio is adequate. and the movement to non-par is positive for margins. But further loss of market share would occur unless LIC pushes up growth rates to match rivals. It’s hard to estimate EV trends. Valuations are cheap which leaves room for some upside.


      • 2.
        Read the given passage and answer the question that follows:
        Biological classification of plants and animals was first proposed by Aristotle on the basis of simple morphological characters. Linnaeus later classified all living organisms into two kingdoms- Plantae and Animalia. Whittaker proposed an claborate five kingdom classification- Monera, Protista, Fungi, Plantae and Animalia. The main criteria of the five kingdom classification were cell structure, body organisation, mode of nutrition and reproduction. and phylogenetic relationships. 
        In the five kingdom classification. bacteria are included in Kingdom Monera. Bacteria are cosmopolitan in distribution. These organisms show the most extensive metabolic diversity. Bacteria may be autotrophic or heterotrophic in their mode of nutrition. Kingdom Protista includes all single-celled eukaryotes such as Chrysophytes. Dinoflagellates. Euglenoids. Slime-moulds and Protozoans. Protists have defined nucleus and other membrane bound organelles. They reproduce both asexually and sexually. Members of Kingdom Fungi show a great diversity in structures and habitat. Most Fungi are saprophytic in their mode of nutrition. They show asexual and sexual reproduction. Phycomycetes. Ascomycetes. Basidiomycetes and Deuteromycetes are the four classes under this kingdom. The plantae includes all eukaryotic chlorophyll-containing organisms. Algae. bryophytes. pteridophytes. gymnosperms and angiosperms are included in this group. The life cycle of plants exhibit alternation of generations- gametophytic and sporophytic generations. The heterotrophic eukaryotic, multicellular organisms lacking a cell wall are included in the kingdom Animalia. The mode of nutrition of these organisms is holozoic. They reproduce mostly by the sexual mode. Some acellular organisms like viruses and viroids as well as the lichens are not included in the five kingdom system of classification.


          • 3.
            Read the given passage and answer the questions that follow:
            Drug major Novartis is planning to divest its Indian eyecare portfolio to Mumbaibased JB Chemicals, in a deal estimated to be around ¥ 1.000 crore. The Novartis spin-off will join the list of several MNCs pruning their drug portfolios and reducing exposure to the Indian market, due to multiple reasons. including increased competition and tough business environment. Sources say the move fits well with the MNC’s strategy to capitalize its ophthalmic therapy, while for JB Chemicals it will offer an entry into a growing business segment.
            Over last few months, global Big Pharma have been divesting their branded generic portfolio to domestic companies and rationalizing their portfolio by selling off key assets. Further, large Indian players are doubling down on India as an attractive diversification from a USgenerics market beaten up heavily by price erosion. As a consequence, several deals were inked where Indian companies snapped up high-growth brands from MNCs and local sellers at attractive valuations. The deal is expected to be announced over the next few days. Emails sent across to Novartis and JB Chemicals did not elicit a response. Ageing population and increasing access to eyecare. especially in emerging markets, is a strong growth opportunity for drug companies. The demand for eyecare is expected to increase significantly as people spend more time in front of tablets and mobile devices. Sources added that existing eyecare portfolio in India of Novartis is understood to be around Z 400-500 crore. including certain brands transferred from eyecare biggie Alcon. when it was spun off from Novartis. In 2019 under a global restructuring move, Novartis had spun off Alcon into a standalone business to focus on its core area of pharmaceuticals. Alcon is a global leader in eyecare, offering solutions to issues like cataracts, glaucoma., retinal diseases and refractive errors. 
            The stock market seems to have got a wind of the potential deal. Over the last few days. scrips of both Novartis India and JB Chem have witnessed a spurt. On December 7, JB Chem traded on a new 52-week high at Z 1555, while Novartis India stock closed Z 706 on Friday. Further. Novartis had last year announced the transfer of sales & distribution of a few of its established medicines, including the Voveran and Calcium range to Dr.Reddy’s.


              • 4.
                In a certain code, O is written as E, A as C, M as I, P as N, and C as S. How will COMPA be written in that code?

                  • SMINC
                  • SETAC
                  • SEINC
                  • SEIMI

                • 5.

                  Spot the error in the given sentence: 
                  \(\underline{\text{He and the other}}\) \(\underline{\text{members of the group}} \underline{\text{spoke after}} \underline{\text{their final victory}}. \)

                    • He and the other
                    • members of the group
                    • spoke after
                    • their final victory.

                  • 6.
                    In a certain code, STOVE is written as FNBLK. Then how will VOTES be written in the same code?

                      • FLKBN
                      • LBNKF
                      • LKNBF
                      • LNBKF

                    Fees Structure

                    Structure based on different categories

                    CategoriesState
                    General2000
                    sc1000
                    pwd1000

                    In case of any inaccuracy, Notify Us! 

                    Comments


                    No Comments To Show